Sunteți pe pagina 1din 135

Assignment Set- 1 Master of Business Administration-MBA Semester 1

MB0038 - Management Process and Organization Behavior Q.1 Write a note on the managerial roles and skills? ANS: According to Mintzberg (1973), managerial roles are as follows: 1. Informational roles 2. Decisional roles 3. Interpersonal roles 1. Informational roles: This involves the role of assimilating and disseminating information as and when required. Following are the main sub-roles, which managers often perform: a. Monitor - collecting information from organizations, both from inside and outside of the organization b. Disseminator - communicating information to organizational members c. Spokesperson - representing the organization to outsiders 2. Decisional roles: It involves decision making. Again, this role can be subdivided in to the following: a. Entrepreneur - initiating new ideas to improve organizational performance b. Disturbance handlers - taking corrective action to cope with adverse situation c. Resource allocators - allocating human, physical, and monetary resources d. Negotiator - negotiating with trade unions, or any other stakeholders

3. Inter`personal roles: This role involves activities with people working in the organization. This is supportive role for informational and decisional roles. Interpersonal roles can be categorized under three sub-headings: a. Figurehead - Ceremonial and symbolic role b. Leadership - leading organization in terms of recruiting, motivating etc. c. Liaison - liasoning with external bodies and public relations activities.

Management Skills: Katz (1974) has identified three essential management skills: technical, human, and conceptual.

Technical skills: The ability is to apply specialized knowledge or expertise. All jobs require some specialized expertise, and many people develop their technical skills on the job. Vocational and on-the-job training programs can be used to develop this type of skill. Human Skill: This is the ability to work with, understand and motivate other people (both individually and a group). This requires sensitivity towards others issues and concerns. People, who are proficient in technical skill, but not with interpersonal skills, may face difficulty to manage their subordinates. To acquire the Human Skill, it is pertinent to recognize the feelings and sentiments of others, ability to motivate others even in adverse situation, and communicate own feelings to others in a positive and inspiring way. Conceptual Skill: This is an ability to critically analyze, diagnose a situation and forward a feasible solution. It requires creative thinking, generating options and choosing the best available option.

Q.2 Explain the social learning theory in details? ANS: One of the most influential learning theories, the Social Learning Theory (SLT), was formulated by Albert Bandura. It encompasses concepts of traditional learning theory and the operant conditioning of B.F. Skinner. However, the theory strongly implies that there are types of learning wherein direct reinforcement is not the causal mechanism; rather, the so called social element can result to the development of new learning among individuals. Social Learning Theory has been useful in explaining how people can learn new things and develop new behaviors by observing other people. It is to assume, therefore, that Social Learning Theory is concerned on observational learning process among people.

A. Basic Concepts 1. Observational LearningThe Social Learning Theory says that people can learn by watching other people perform the behavior. Observational learning explains the nature of children to learn behaviors by watching the behavior of the people around them, and eventually, imitating them. With the Bobo Doll experiment(s), Bandura included an adult who is tasked to act aggressively toward a Bobo Doll while the children observe him. Later, Bandura let the children play inside a room with the Bobo Doll. He affirmed that these children imitated the aggressive behavior toward the doll, which they had observed earlier. After his studies, Bandura was able to determine 3 basic models of observational learning, which include:

a. A Live Model, which includes an actual person performing a behavior. b. A Verbal Instruction Model, which involves telling of details and descriptions of a behavior. c. A Symbolic Model, which includes either a real or fictional character demonstrating the behavior via movies, books, television, radio, online media and other media sources. 2. The state of mind (mental states) is crucial to learning.In this concept, Bandura stated that not only external reinforcement or factors can affect learning and behavior. There is also what he called intrinsic reinforcement, which is in a form of internal reward or a better feeling after performing the behavior (e.g. sense of accomplishment, confidence, satisfaction, etc.) 3. Learning does not mean that there will be a change in the behavior of an individual.

B. Modeling Process The Modeling Process developed by Bandura helps us understand that not all observed behaviors could be learned effectively, nor learning can necessarily result to behavioral changes. The modeling process includes the following steps in order for us to determine whether social learning is successful or not:

Step 1: Attention Social Cognitive Theory implies that you must pay attention for you to learn. If you want to learn from the behavior of the model (the person that demonstrates the behavior), then you should eliminate anything that catches your attention other than him. Also, the more interesting the model is, the more likely you are to pay full attention to him and learn.

Step 2: Retention Retention of the newly learned behavior is necessary. Without it, learning of the behavior would not be established, and you might need to get back to observing the model again since you were not able to store information about the behavior.

Step 3: Reproduction When you are successful in paying attention and retaining relevant information, this step requires you to demonstrate the behavior. In this phase, practice of the behavior by repeatedly doing it is important for improvement. Step 4: Motivation Feeling motivated to repeat the behavior is what you need in order to keep on performing it. This is where reinforcement and punishment come in. You can be rewarded by demonstrating the behavior properly, and punished by displaying it inappropriately

Q.3 Explain the Big 5 model of personality? Ans : The big five model of personality is designed to bring out behaviors an individual expresses in his dealings with people as well as in their response to changes in circumstances as well as the environment. Generally, these five factors of personality are used to come up with a description of the human personality. In this text, I explain Tishas findings as well as look at how the predictions could be used to come up with a prediction of her success as a manager. The big five model of personality

Openness to experience This factor acts to distinguish conventional individuals from those who are in one way or the other imaginative. Traits linked to individuals who are open to experience include intellectual curiosity, sensitivity to beauty as well as responsiveness to art. According to Cattell, H.E.P et al. (2007), such people are very creative and they are very likely to be more accommodating to beliefs considered unconventional. Such people are hence more likely to be accommodating to the views of others in the organizational setting and they are more likely to encourage innovation at the workplace. Tishas high score shows that she will be more straightforward and more likely see the simple aspects of complex situations.

Conscientiousness This factor shows an individuals ability and drive to achieve goals and strive for achievement. It also indicates that an individual has self discipline and prefers planned behavior as opposed to spontaneous behavior. Grucza et al. (2007) notes that individuals who score highly in conscientiousness have a good impulse control and tend to be goal oriented. Tishas very high performance in Conscientiousness shows that she is organized and very attentive to detail. In an organizational setting, Tisha would be more likely to strive to ensure that organizational goals and objectives are met.

Extraversion This factor consists of emotions that can be said to be positive as well as an industrious personality and ability to relate well with others. Cattell, H.E.P et al. (2007) notes that extraverts enjoy other peoples company and tend to be very active in group efforts. Tishas high score in extraversion indicates that in the work setting as a manager, she would tend to encourage team work. She will also dominate board and other meetings and have a high chance of pushing her proposals and suggestions forward.

Agreeableness This factor is characterized by compassion and cooperation towards others as opposed to antagonism tendencies. Cattell, H.E.P et al. (2007) notes that such

individuals are easy to get along with and are more likely to be very accommodating to the views of others. Tishas moderately high score on this factor shows that though she will be accommodate others view, she will expect her views also to be taken into consideration. She is likely to trust her subordinates more.

Neuroticism This factor involves a high likelihood to get angry and have other negative emotions like anxiety. In most cases, this characteristic is linked to emotional instability. Individuals scoring low on neuroticism will experience bouts of mood swings triggered by frustrations by minor issues at the workplace (Grucza et al. 2007). Tishas low score on this factor shows she will be more likely to cope with problems as a manager and she will tend to be calmer while handling difficult situations.

Q.4 What are the different factors influencing perception? Ans : Perception is our sensory experience of the world around us and involves both the recognition of environmental stimuli and action in response to these stimuli. Through the perceptual process, we gain information about properties and elements of the environment that are critical to our survival.

A number of factors operate to shape and sometimes distort perception These factors can reside: i) In the perceiver

ii) In the Object or target being perceived or iii) In the context of the situation in which the perception is made. 1. Characteristics of the Perceiver: Several characteristics of the perceiver can affect perception. When an individual looks at a target and attempts to interpret what he or she stands for, that interpretation is heavily influenced by personal characteristics of the individual perceiver. The major characteristics of the perceiver influencing perception are: a) Attitudes: The perciver's attitudes affect perception. For example, Mr. X is interviewing candidates for a very important position in his organization - a position that requires negotiating contracts with suppliers, most of whom are male. Mr. X may feel that women are not capable of holding their own in tough negotiations. This attitude with doubtless affect his perceptions of the female candidates he interviews. b) Moods: Moods can have a strong influence on the way we perceive someone. We think differently when we are happy than we do when we are depressed. In addition, we remember information that is consistent with our mood state better than information that is inconsistent with our mood state. When in a positive mood, we form more positive impressions of other. When in a negative mood, we tend to evaluate others unfavourably. c) Motives: Unsatisfied needs or motives stimulate individuals and may exert a strong influence on their perceptions. For example, in an organizational context, a boss who is insecure perceives a sub ordinate's efforts to do an outstanding job as a threat to his or her own position. Personal insecurity can be translated into the perception that others are out to "get my job", regardless of the intention of the subordinates.

d) Self - Concept: Another factor that can affect social perception is the perceivers self-concept. An individual with a positive self-concept tends to notice positive attributes in another person. In contrast, a negative self-concept can lead a perceiver to pick out negative traits in another person. Greater understanding of self allows us to have more accurate perceptions of others.

e) Interest: The focus of our attention appears to be influenced by our interests. Because our individual interests differ considerably, what one person notices in a situation can differ from what other perceive. For example, the supervisor who has just been reprimanded by his boss for coming late is more likely to notice his colleagues coming late tomorrow than he did last week. f) Cognitive structure: Cognitive structure, an individual's pattern of thinking, also affects perception. Some people have a tendency to perceive physical traits, such as height, weight, and appearance, more readily. Cognitive complexity allows a person to perceive multiple characteristics of another person rather than attending to just a few traits. g) Expectations: Finally, expectations can distort your perceptions in that you will see what you expect to see. The research findings of the study conducted by Sheldon S Zalking and Timothy W Costello on some specific characteristics of the perceiver reveal i) Knowing oneself makes it easier to see others accurately. ii) One's own characteristics affect the characteristics one is likely to see in other. iii) People who accept themselves are more likely to be able to see favourable aspects of other people. iv) Accuracy in perceiving others is not a single skill. These four characteristics greatly influence how a person perceives other int he

environmental situation.

2) Characteristics of the Target : Characteristics in the target that is being observed can affect what is perceived. Physical appearance pals a big role in our perception of others. Extremely attractive or unattractive individuals are more likely to be noticed in a group than ordinary looking individuals. Motions, sound, size and other attributes of a target shape the way we see it. Verbal Communication from targets also affects our perception of them. Nonverbal communication conveys a great deal of information about the target. The perceiver deciphers eye contact, facial expressions, body movements, and posture all in a attempt to form an impression of the target.

3) Characteristics of the Situation: The situation in which the interaction between the perceiver and the target takes place, has an influence on the perceiver's impression of the target. The strength of the situational cues also affects social perception. Some situations provide strong cues as to appropriate behaviour. In this situation, we assume that + i.e individual's behaviours can be accounted for by the situation, and that it may not reflect the individual's disposition.

Q.5 Write a note on contemporary work cohort? ANS: Contemporary Work Cohort Contemporary Work Cohort, proposed by Robbins (2003) divides the work force into different groups depending on the era or period in which they have entered

into work. It stresses upon individuals values which reflect the societal values of the period in which they grew up. The cohorts and the respective values have been listed below: 1.VeteransWorkers who entered the workforce from the early 1940s through the early 1960s and exhibited the following value orientations: 1. They were influenced by the Great Depression and World War II 2. Believed in hard work 3. Tended to be loyal to their employer 4. Terminal values: Comfortable life and family security 2.BoomersEmployees who entered the workforce during the 1960s through the mid1980s belonged to this category and their value orientations were: a. Influenced heavily by John F. Kennedy, the civil rights and feminist movements, the Beatles, the Vietnam War, and baby boom competition. b. Distrusted authority, but gave a high emphasis on achievement and material success. c. Organizations who employed them were vehicles for their careers. d. Terminal values: sense of accomplishment and social recognition.

3.Xersbegan to enter the workforce from the mid1980s.

They cherished the following values: a. Shaped by globalization, two career parents, MTV, AIDS, and computers. b. Value flexibility, life options, and achievement of job satisfaction. c. Family and relationships were important and enjoyed team oriented work. d. Less willing to make personal sacrifices for employers than previous generations. e. Terminal values: true friendship, happiness, and pleasure

4.Ne xte rsmost recent entrants into the workforce. a. Grew up in prosperous times, have high expectation, believe in themselves, and confident in their ability to succeed. b. Never ending search for ideal job; see nothing wrong with job hopping. c. Seek financial success. d. Enjoy team work, but are highly self reliant. e. Terminal values: freedom and comfortable life. Q.6 What are the special issues in motivation? Discuss

Ans : Some of the special issues in motivation are discussed below. Motivating Professionals The professional employees likely to seek more intrinsic satisfaction from their work than blue-collar employees. They generally have strong and long term commitments to their field of expertise are perhaps more loyal to their profession than to their employer. They need to regularly update their knowledge, and their commitment to their profession. Therefore, extrinsic factors such as money and promotions would be low on their priority list. Rather, job challenge tends to be ranked high. They like to tackle problems and find solutions.

Managerial Implications: Provide them with ongoing challenging projects. Give them autonomy to follow their interests and allow them to structure their work. Reward them with educational opportunities. Also reward them with recognition.

Motivating temporary Workers: Temporary workers may be motivated if: They are provided with permanent job opportunity The opportunity for training is provided to them Provide equitable pay. Motivating Low Skilled Service Workers: One of the most challenging managerial tasks in to motivate low skilled workers who are involved in repetitive physical work, where higher education and skills are not required. For this category of people, flexible work schedules and higher pay package may be proved effective motivational factors. Motivating Low-Skilled Service Workers involves: Recruit widely. Increase pay and benefits. Make jobs more appealing.

Motivating People Doing Highly Repetitive Tasks: Recruit and select employees that fit the job. Create a pleasant work environment. Mechanize the most distasteful aspects of the job.

Assignment Set- 2 Q.1 Explain the theories of emotion? ANS: Psychologists have proposed a number of theories about the origins and function of emotions. The theorists behind the dissenting views do agree on one thing, however: emotion has a biological basis. This is evidenced by the fact that the amygdala (part of the limbic system of the brain), which plays a large role in emotion, is activated before any direct involvement of the cerebral cortex (where memory, awareness, and conscious "thinking" take place). In the history of emotion theory, four major explanations for the complex mental and physical experiences that we call "feelings" have been put forward. They are: the James-Lange theory in the 1920's, the Cannon-Bard theory in the 1930's, the Schacter-Singer theory in the 1960's, and most recently the Lazarus theory, developed in the 1980's and 90's.

The James-Lange Theory The James-Lange theory proposes that an event or stimulus causes a physiological arousal without any interpretation or conscious thought, and you experience the resulting emotion only after you interpret the physical response. Example: You're late leaving work, and as you head across the parking lot to your car, you hear footsteps behind you in the dark. Your heart pounds and your hands start to shake. You interpret these physical responses as fear.

The Cannon-Bard Theory The Cannon-Bard theory, on the other hand, suggests that the given stimulus evokes both a physiological and an emotional response simultaneously, and that neither one causes the other.

Example: You're home alone and hear creaking in the hallway outside your room. You begin to tremble and sweat, and you feel afraid.

The Schacter-Singer Theory The Schachter-Singer theory takes a more cognitive approach to the issue. Schacter and Singer believe that an event causes physiological arousal, but that you must then identify a reason for the arousal before you label the emotion. Example: You're taking the last bus of the night, and you're the only passenger. A single man gets on and sits in the row behind you. When your stop comes around, he also gets off the bus. He's walking behind you. You feel tingles down your spine with a rush of adrenaline. You know that there have been several muggings in your city over the past few weeks, so you feel afraid.

The Lazarus Theory The Lazarus theory builds on the Schacter-Singer theory, taking it to another level. It proposes that when an event occurs, a cognitive appraisal is made (either consciously or subconsciously), and based on the result of that appraisal, an emotion and physiological response follow. Example: You're buying a few last-minute items at the gas station, when two young men in hooded sweatshirts enter the store in a hurry, with their hands in their jacket pockets. You think perhaps they're here to rob the place, so you get scared, and your feel like you might throw up. While each of these theories is based in research, there is no absolute proof as yet how emotions arise in our bodies and minds, or what determines our own individual experiences of them. What we do know is that feelings are a powerful force to be reckoned with, and should never be belittled. Q.2 Discuss the techniques of decision making in

groups? ANS : Planning for Decision Making While decision making without planning is fairly common, it is often not pretty. The terms used to describe it--crisis management, putting out fires, seat-of-thepants governing--all reveal the inelegance and awkwardness of this way of life. Planning allows decisions to be made in a much more comfortable and intelligent way. Planning even makes decisions easier by providing guidelines and goals for

the decision. We might even say that planning is a type of decision simplification technique (see the discussion of these techniques below). Decision makers will find four major benefits to planning: 1. Planning allows the establishment of independent goals. The vision which will shape the decisions is set apart from surrounding events. Decisions are not made only as reactions to external stimuli. "Management by firefighting" is replaced by a conscious and directed series of choices. Managers now steer the organization, individuals now steer their lives, rather than being steered by external forces. Sometimes the difference between planning and not planning is described as "proactive" (taking control of the situation) versus "reactive" (responding to stimuli). 2. Planning provides a standard of measurement. A plan provides something to measure against, so that you can discover whether or not you are achieving or heading toward your goals. As the proverb says, If you don't know where you're going, it doesn't matter which way you go. 3. Planning converts values to action. When you are faced with a decision, you can consult your plan and determine which decision will help advance your plan best. Decisions made under the guidance of planning can work together in a coherent way to advance company or individual goals. Planning is useful in emergency situations, too. When a crisis arises, a little thought about the overall plan will help determine which decision to make that will not only help resolve the crisis but will also help advance the overall plan. Without a plan, crises are dealt with haphazardly and decisions are made which may ultimately be in conflict with each other.

4. Planning allows limited resources to be committed in an orderly way. Budgets, time, effort, manpower--all are limited. Their best use can be made when a plan governs their use. A simple example would be planning to buy a house or a car. Rather than having to decide between buying the item right now with all cash or never having it, you can plan to buy it over several years by making payments. Or, you might combine this plan with the plan to buy a smaller house and add rooms later as they could be afforded. By planning you can thus accomplish things that might otherwise look impossible. Decision Levels We all recognize that some decisions are more important than others, whether in their immediate impact or long term significance. As a means of understanding the significance of a decision so that we can know how much time and resources to spend on it, three levels of decision have been identified: 1. Strategic. Strategic decisions are the highest level. Here a decision concerns general direction, long term goals, philosophies and values. These decisions are the least structured and most imaginative; they are the most risky and of the most uncertain outcome, partly because they reach so far into the future and partly because they are of such importance. For example: Decisions about what to do with your life, what to learn, or what methods to use to gain knowledge (travel, work, school) would be strategic. Whether to produce a low priced product and gain market share or

produce a high priced product for a niche market would be a strategic decision.

2. Tactical. Tactical decisions support strategic decisions. They tend to be medium range, medium significance, with moderate consequences. For example: If your strategic decision were to become a forest ranger, a tactical decision would include where to go to school and what books to read. Or if your company decided to produce a low priced product, a tactical decision might be to build a new factory to produce them at a low manufacturing cost. 3. Operational. These are every day decisions, used to support tactical decisions. They are often made with little thought and are structured. Their impact is immediate, short term, short range, and usually low cost. The consequences of a bad operational decision will be minimal, although a series of bad or sloppy operational decisions can cause harm. Operational decisions can be preprogrammed, pre-made, or set out clearly in policy manuals. For example: If your tactical decision is to read some books on forestry, your operational decision would involve where to shop for the books. You might have a personal policy of shopping for books at a certain store or two. Thus, the operational decision is highly structured: "Whenever books are needed, look at Joe's Books." An important comment should be made here. Issues should be examined and decisions should be made at all of these levels. If you discover that nearly all of your thinking and decision making is taking place at the operational level, then you are probably not doing enough strategic thinking and planning. As a result you will lead a reactive life, responding only to the forces around you and never getting control of your life, your direction or your goals.

Some Techniques for Decision Making

This is a list of easy, practical techniques that can be applied to simple or complex decisions. They share the assumption that circumspect analysis is the key to making good decisions. Many decisions are made with too little information and too little thought, in a non-deliberate way. Think about it for a moment: how many people do you know who commonly spend even five minutes structuring and analyzing a decision? Note how these techniques provide a visible, structured, orderly set of factors involved in a decision, so that the decision maker can consider them in a thoughtful and coherent way. The first three techniques are especially for whether-type decisions, those involving yes/no, either/or, or two-possibility decisions. 1. T-Chart. A T-Chart is an orderly, graphic representation of alternative features or points involved in a decision. In one form, it can be a list of positive and negative attributes surrounding a particular choice. Drawing up such a chart insures that both the positive and negative aspects of each direction or decision will be taken into account. For example, what are the pros and cons of deciding to buy a sport utility vehicle? PRO better visibility safer structure can take off road CON higher insurance poorer gas mileage more expensive maintenance

In another form, two possible choices are listed, with the good points or arguments or effects listed for each. Suppose your company is trying to decide whether to create its own advertising or hire an agency.

Use Outside Agency professional work expertise of ideas media connections

Write Ads In-House faster product better knowledge of product use same ad in flyers

To fill out this latter form, more than two choices can be included, and a list of negatives for each choice can be added as well. 2. PMI. Edward de Bono refines the T-Chart idea into a three part structure, which he calls PMI for plus, minus, and interesting. Here you first list all the plus or good points of the idea, then all the minus or bad points, and finally all the interesting points--consequences, areas of curiosity or uncertainty, or attributes that you simply don't care to view as either good or bad at this point (consequences that some people might view as good and others might view as bad, for example). The "interesting" category also allows exploration of the idea or choice outside the context of judgment--you don't have to evaluate the attribute into a positive or negative category.

As simple as this technique seems to be, and as often as others will tell you, "Well, of course, everyone does that all the time," this is a very powerful but much neglected technique. Most people believe they list the pluses and minuses of a decision before making it, but in actual practice, many people make a decision or form an opinion before they consider the evidence in an orderly way. Only after they make a decision do they hunt around for reasons to support it. Considering the evidence on both (or all) sides before you commit yourself emotionally and psychologically to a position will have a major impact on the quality of your decision making. 3. Buriden's Ass. This method of decision making is used when two or more equally attractive alternatives are faced. (From an old fable of an ass placed between two equally nice bales of hay. The ass couldn't decide which bale to turn to because they were both so attractive, and so it starved to death from indecision.) The method is simply to list all the negative points or drawbacks about each decision. That is, when two or more alternatives seem very desirable, we become blinded to any drawbacks. The Buriden's Ass method simply focuses on the drawbacks. 4. Measured Criteria. With this technique, you list the criteria you want your decision to meet and assign points to each criterion based on its relative importance in the decision. Then, each alternative is given a certain number of points according to how fully it meets the criterion. For points you can use a scale of 1 to 10, 1 to 100, or any other range that makes sense to you. In the example below, traveling by train is rated at 25 out of 30 points for the "comfort" criterion, while the plane is ranked a little less comfortable, at 21 out of

30. Once all the alternatives have been assigned their due points for each criterion, all the points for each alternative are added up and the alternative with the highest total points is the one chosen. In the example below, that would be the plane. 5. Decision Matrix or Weighted Decision Table. This is a slightly more sophisticated version of the measured criteria technique. Here a table is set up with each criterion given a weight depending on its importance in the decision and with each alternative given a ranking for that criterion. Q.3 Elaborate the different stages in process of conflict? ANS: The conflict process can be seen as comprising five stages: (1) Potential opposition or incompatibility- The first step in the conflict process is the presence on conditions that create opportunities for conflict to rise. These cause or create opportunities for conflict to rise. (2) Cognition and personalization -conflict must be perceived by the parties to it whether or not conflict exists is a perception issue. (3) Intentions -Intentions are decisions to act in a given way intentions intervene between peoples perception and emotions and their overt behavior. These intentions are Competing, Collaborating, Avoiding, Accommodating or Compromising. (4) Behavior -This is a stage where conflict becomes visible. The behavior stage includes the statements, actions and reactions made by the conflicting parties. (5) Outcome- The action reaction interplay between the conflicting parties result in consequences. These outcomes may be functional in that the conflict results in an improvement in the groups performance, or dysfunctional in that it hinders group performance.

Q.4 Write a note on GAS ( General Adaptation Syndrome)? ANS: GAS or General Adaptation Syndrome is a very common medical problem that can have very serious repercussions if left unattended. It was psychologist Hans Selye who discovered that prolonged and excessive stress can lead to infection, illness, disease and death. He then named this condition General Adaptation Syndrome (GAS). In general, there are three stages of GAS. They are Alarm, Resistance and Exhaustion. Let's talk a bit about each stage. Stage One: Alarm Alarm is essentially the initiation of the "fight of flight" response which prepares the body for life-threatening situations. Normal body activities such as the digestive and immune functions are blunted or nearly shut down in order to supply resources to more immediate muscular and emotional needs. Stage Two: Resistance Now, if we're under a constant state of alarm, we are in essence living in constant state of stress. At this point we start becoming used to these stress levels. The funny thing about this stage is that initially our body and immune system are becoming more resistance to stress and disease. However, this stage requires abnormally high levels of emotional and physical resources. If things do not change for the better then the next stage of the GAS process is inevitable.

Stage Three: Exhaustion Eventually reality kicks in and our bodies are unable to maintain high levels of stress resistance. Parts of the body literally start to break down and we become very unwell. To conclude this first section, please understand that Hans Selye and others experts are convinced that out-of-control stress negatively influences a person's entire organism. Furthermore, these authorities believe that if left untreated, run away stress can result in disease and eventual death. The point here is that as a hypnotist, your contribution as a stress management consultant is far more important than most people realize. Unmanaged stress is not just an 'inconvenience'. It is a health threaten By the way, remember to take some of your own medicine once in a while and listen to stress reduction hypnosis CDs. It is much easier to convince a client of the benefits of successful stress management when you are 100% congruent because of satisfying experience. By the way, remember to take some of your own medicine once in a while and listen to stress reduction hypnosis CDs. It is much easier to convince a client of the benefits of successful stress management when you are 100% congruent because of satisfying experience. Now, let's look at a case history that shows how to help a client suffering from severe GAS. When You're Client Has GAS --The Case of Bob First of all, in order to help a client with G.A.S., they must learn how to F.A.R.T. and B.U.R.P. so they can ultimately P.O.O.P.

Now, before a person can B.U.R.P. (Begin Utilizing Response Procedures) or P.O.O.P. (Pursue Optimal Outcomes Persistently), they have to F.A.R.T. That is, they have to first Formulate Appropriate Response Techniques. This can be reduced to a simple statement. When a person starts to suffer from stress, they must S.T.O.P. and B.E.A.N.O. That is, they must Start To Observe Purposefully so that they can then Become Excited And Noticeably Optimistic. If they are unable to S.T.O.P and B.E.A.N.O., then they will have to F.A.R.T., B.U.R.P. and most likely, P.O.O.P. Let's look at a case history involving my client Bob. Bob has suffered for quite some time from severe stress effects. When he first came to our office, we realized that he was emotionally plugged up. In other words, his feelings were so bottled up that he absolutely could not P.O.O.P. The reason became apparent when it was discovered that he had never before learned how to properly F.A.R.T. and B.U.R.P. in response to stress. I knew this was not a case where the client could simply B.E.A.N.O. because that would not address the cause of his inability to P.O.O.P. What he needed was an E.N.E.M.A. (Entirely New Explicit Meaning Association). As the reader may have guessed, this was a very sensitive situation because it involved reframing a learned response that was created in response to a family of origin dynamic.

Apparently there was some S.O.B. (Subtle Obfuscating Behavior) responsible for the onset of the client's difficulty. Defeating this without running into family loyalty conflicts and their accompanying resistance effects is quite a job that requires deft handling and discernment. However, since the client was extremely motivated to get past his problem and move on we advanced a rather bold tactic. We decided to initiate a reverse E.N.E.M.A. The client became quite excited at the idea. This was because we explained to him that we would go back in time and teach him as a young child how to P.O.O.P. In summary, the client has been relieved of his GAS because after undergoing a reverse E.N.E.M.A. he was able to B.E.L.C.H. (Begin Expurgating Lousy Childhood History) which enabled him to successfully F.A.R.T., B.U.R.P. and P.O.O.P. The point to this second section is that sometimes to heal, we just need to stop hanging onto old 'stuff' from our past because doing so makes us much more prone to insidious medical disorders such as General Adaptation Syndrome. One must wonder that if GAS can cause such serious consequences then, what common diseases are actually the result of prolonged, unhealed stress? Finally, as you can probably tell, this article was meant to be tongue-in-cheek but the point is very serious. Helping a client reduce stress is not just a "mental massage" that feels good in the short term. It is an essential element of a good and happy life.

And, don't forget that the family members of your client are also benefiting from your valuable and skilled assistance because they have longer to enjoy a healthier and happier mom, dad or other important person. In closing I'd like to share a quote that can point a person toward a significantly more relaxed state of mind and being: Q.5 Discuss the power and influence tactics? ANS: The general objective of this study was to examine the supervisors and subordinates use of power and their relationships to supervisors use of influence tactics. Specifically, the purpose of this study was to examine power congruence and its impact on influence tactics in manufacturing companies in Malaysia. The present research differs from the previous studies by linking power congruence between supervisors and subordinates power either from self or as perceived by their subordinates or supervisors with three dimensions of influence tactics known as, hard, soft, and rational appeal tactics. This study is perhaps the first that tested congruence hypothesis in leadership framework. The objective was to gain insight into ways by which the management of manufacturing companies might use their power to enhance the effective use of influence tactics on their subordinates. Ten broadly hypothesized relationships were tested in a field study with a sample of 385 pairs of supervisors and subordinates working in 82 manufacturing companies in Selangor/Kuala Lumpur, Penang, and Sarawak. Data were gathered from both supervisors and their subordinates by means of questionnaires. Methodologically, past research had been prone to common method bias. However, this study has demonstrated to be relatively free from this bias by collecting data from two sources. By and large, the results from

the analyses have indicated moderate support for the hypotheses. This study is perhaps the first to generate a new set of power congruence items in which simultaneous measurement from two perspectives-supervisors and subordinates-were taken to examine the aspect of mutuality. The first four hypotheses which investigate the direct relationship between supervisors or subordinates power and influence tactics revealed that supervisors would apply various influence tactics on their subordinates. Rational appeal tactics has exhibited the highest mean as compared with soft and hard influence tactics in the direct relationship between power and influence tactics. For the indirect hypotheses, only one particular dimension of influence tactics was found significant for each power congruence hypotheses. The results confirmed that when both supervisors and subordinates were perceived to have position power, the use of hard influence tactics was most apparent. Conversely, when both of them were seen to have personal power, supervisors would resort to the use of soft influence tactics. Inevitably, this study provides a conceptual foundation for the effective use of influence tactics. This study may be useful for those who are in positions of influence, to help the supervisors and subordinates understand more clearly the bases of their own actions, and the possible alternatives to their actions. Practically, this research points to the fact that Malaysian managers and executives needto be trained in the effective use of influence tactics. Q.6 Explain the characteristics of organization Development? ANS: Values of OD This model places human centered values above everything else. They are the engine of its success. These values include mutual trust and confidence, honesty and open communication, sensitivity and to the feeling and

emotions of others, shared goals, and a commitment to addressing and resolving conflict (Buchanan and Huczynski, 1997, p.489). There are organizations who value these human attributes above all other quick fix benefits. Stephen Robbins (1986, p.461) expresses these human values more concisely as follows: a) The individual should be treated with respect and dignity. b) The organization climate should be characterized by trust, openness and support. c) Hierarchical authority and control are not regarded as effective mechanisms. d) Problems and conflicts should be confronted, and not disguised or avoided. e) People affected by change should be involved in its implementation. These values make OD a unique model and it will be shown later why it is the most suitable approach in resolving human afflictions and problems such as poverty alleviation. OD Approaches There are at least three planned change models that have been identified by Cummings and Worley (1997) as shown in figure II below. The first and the second are principally different but the third one is an improved version of Action Research methodology which has taken centre stage in the 1990s to take care of the trend of emerging mega organizations, strategic alliances, mergers and public private partnerships. The improved version named the Contemporary Action Research model is of great interest to the workshop participants as it can be used to promote projects in both public and private organizations. Later in this paper it will be shown how it can be used to promote Poverty Reduction Strategies and gender mainstreaming in local government.

The distinction between Lewins model and Action Research model is in the repetitive nature of action research. Lewins model is a once only intervention. Buchanan and Huczynski, (1997) give seven steps that are followed in this intervention. 1. Scouting - the initial stages of consultant and client exchanging ideas on the problem and the appropriate approach. 2. Entry - contract is entered into to formalize the relationship of consultant and client. 3. Diagnosis - information gathering to define the problem and identify causes. 4. Planning - consultant and client members jointly establish the goals of an OD intervention and the proposed approach 5. Action - the intervention strategies are implemented. 6. Stabilization and Evaluation - the change is stabilized (freezing takes place) and the outcomes are assessed. 7. Termination - The consultant withdraws from the assignment. The action research model differs in several particular ways with the Lewins model. In action research the outcomes are fed back so that further improvements and changes can be made. This distinguishes this model as a cyclical and iterative process. The research aspect connotes a search 5 for knowledge that may be used elsewhere. OD Toolkits The above outlined procedure is the process through which results area achieved but action research methodology has what are described as toolkits. These are used to address specific areas of problem. Once diagnosis is complete and the problem

identified one or more of these kits or interventions are employed to solve the problem. Some of these include; process consultation, change the structure, survey feedback, team building, inter-group development, role negotiation and sensitivity training. These are just a number but action research allows flexibility for the consultant to formulate a package of his own interventions to suit the clients 6 needs. Such flexibility was illustrated through an OD consultancy in a local government organization.

Organizational Development Interventions The Existence of certain conditions in an organization which may be described as the internal environment is sometimes assumed. Those who have been in an organization for too long cease to be conscious of such an environment and probably only newcomers notice such conditions. Different interventions are used when a need arises to change those conditions. Some of those interventions which are mentioned above deserve further explanation here. Cultural Analysis This is perhaps one of the most complex change action. Corporate culture is the result of long term social learning and constitutes of basic assumptions, values, norms and artifacts that have worked well in an organization. These are passed on to succeeding generations of employees (Cummings and Worley, 1997). Some of these may have arisen from the principles of the founder and subsequently reinforced by succeeding top hierarchies of the organization. They may have an emphasis on product quality, customer care or employee relations. When problems arise the first question which would arise is how do we do things here? The answer

will inform the type of solution that emerges. Therefore the organizational culture influences organizational strategy, performance and policies. To change organizational culture may be a traumatic experience and will require careful analysis and handling. Such is the trauma that was experienced by American companies in the 1980s when they adopted the Japanese approach which was influence by a strong organizational culture of employee participation, open communication, Security and equality.

Process Consultation People conceive consultation as that situation where an expert is invited to advise an organization that is experiencing a problem. Edger Schein, (1998) has distinguished three types of consultations as; the expertise model, the doctor patient model and the process consultation model. The first model assumes that the client purchases from the consultant some expert information or service that he is unable to provide for himself. In the doctor - patient model involves an activity similar to sickness diagnosis. The client who suspects or feels there is something wrong in the organization invites the doctor (consultant) to diagnose the organization so that he can advise on what is not right. The symptoms of the sickness may be low sales, employee instability or falling product quality. The doctor (consultant) diagnoses, prescribes and administers the cure. In this two models the knowledge and expertise remains with the expert and leaves the client fully dependent on the expert for future problems. OD advocates process consultation promoted by Schein who defines the process as: The creation of a relationship with the client that permits the client to perceive, understands, and act on the process events that occur in the clients

internal and external environment in order to improve the situation as defined by the client (Schein, 1998, p.20). Schein argues that both the expert and the doctor models are remedial models while as the process consultation model is both remedial and preventive. The purport of this model is to engage an external consultant on a flexible advisory capacity to work with the clients members in diagnosing the problems, planning the actions and finding the solutions together. This way the consultant helps the organizations individuals to understand internal problems and build capacity to identify appropriate problem - solving action. The consultant need not be an expert in the problem at hand but his expertise is in facilitating a process that carries everybody in the search for solutions. This approach follows the rationale that the answers are with the people. So the only help required is leadership in diagnoses and in the process through the various stages. The purpose also is to ensure that once the consultant leaves the people have the capacity to solve the next round of problems. In the first two models the consultant will have to be recalled each time a new problem arises or an old one recurs.

Structure Change This is an intervention that helps change the structure of an organization to make the work more interesting, challenging or productive. It may involve such activities as job enrichment, job enlargement, formation of autonomous work teams or business re - engineering. Other actions may involve decentralization/centralization in an organization, flattening or extending of an organizations structure or even redesigning of focus from region to product or vice versa.

Team Building Team work is vital to the functioning of modern organizations. Members of teams bring different strings to the group such as leader, investigator, motivator, finisher, clown, coordinator, thinker, negotiator or politician. These roles are used at different stages of production when such role play becomes essential in the groups work. Teams take over from hierarchical systems where individuals are assumed to know everything depending on their level in the authority ladder. This system denies the organization the cumulative advantage of skills and strengths in different individuals.

Role Negotiation A misunderstanding between two individuals in an organization or group can affect its effectiveness. This is usually caused by lack of shared awareness, misunderstanding or lack of trust. This intervention helps to clarify individual perceptions and mutual expectations so that differences can be identified and reconciled or resolved.

Assignment Set- 1

Master of Business Administration - MBA Semester -1

MB0039 - Business Communication

Q.1 Explain the different types of communication with relevant examples? ANS: Communication is a process that involves exchange of information, thoughts, ideas and emotions. Communication is a process that involves a sender who encodes and sends the message, which is then carried via the communication channel to the receiver where the receiver decodes the message, processes the information and sends an appropriate reply via the same communication channel. Types of Communication Communication can occur via various processes and methods and depending on the channel used and the style of communication there can be various types of communication. Types of Communication Based on Communication Channels Based on the channels used for communicating, the process of communication can be broadly classified as verbal communication and non-verbal communication.

Verbal communication includes written and oral communication whereas the nonverbal communication includes body language, facial expressions and visuals diagrams or pictures used for communication. Verbal Communication Verbal communication is further divided into written and oral communication. The oral communication refers to the spoken words in the communication process. Oral communication can either be face-to-face communication or a conversation over the phone or on the voice chat over the Internet. Spoken conversations or dialogs are influenced by voice modulation, pitch, volume and even the speed and clarity of speaking. The other type of verbal communication is written communication. Written communication can be either via snail mail, or email. The effectiveness of written communication depends on the style of writing, vocabulary used, grammar, clarity and precision of language. Nonverbal Communication Non-verbal communication includes the overall body language of the person who is speaking, which will include the body posture, the hand gestures, and overall body movements. The facial expressions also play a major role while communication since the expressions on a persons face say a lot about his/her mood. On the other hand gestures like a handshake, a smile or a hug can independently convey emotions. Non verbal communication can also be in the form of pictorial representations, signboards, or even photographs, sketches and paintings. Types of Communication Based on Style and Purpose Based on the style of communication, there can be two broad categories of communication, which are formal and informal communication that have their own set of characteristic features.

Formal Communication Formal communication includes all the instances where communication has to occur in a set formal format. Typically this can include all sorts of business communication or corporate communication. The style of communication in this form is very formal and official. Official conferences, meetings and written memos and corporate letters are used for communication. Formal communication can also occur between two strangers when they meet for the first time. Hence formal communication is straightforward, official and always precise and has a stringent and rigid tone to it. Informal Communication Informal communication includes instances of free unrestrained communication between people who share a casual rapport with each other. Informal communication requires two people to have a similar wavelength and hence occurs between friends and family. Informal communication does not have any rigid rules and guidelines. Informal conversations need not necessarily have boundaries of time, place or even subjects for that matter since we all know that friendly chats with our loved ones can simply go on and on. Q.2 What are the general principles of writing especially business writing? ANS: The process of good writing involves three basic steps - preparing, writing, and editing. Practicing the following 16 principles will help you be a more effective writer. 1. Know your objective Think before you write. What's your goal? Make sure you fully understand the assignment. Are you writing a one-paragraph executive summary or a five-page

report? Try answering this question: What specifically do I want the reader to know, think, or do? 2. Make a list Write down the ideas or points you want to cover. Why? This helps you get started in identifying the key ideas you want to discuss. If you have trouble getting started, try discussing your ideas with someone else. "Kicking an idea around" often helps you clarify your objective and fine-tune what you are trying to accomplish. 3. Organize your ideas Just as it's difficult to find what you want in a messy, disorganized desk drawer, it's hard to find important ideas in a poorly organized message. Here are a few ways you can organize your ideas:

Importance - Begin with the most important piece of information and then move on to the next most important. Chronological order - Describe what happened first, second, third. Problem-Solution - Define the problem, then describe possible alternatives or the solution you recommend. Question-Answer - State a question and then provide your answer.

Organize your ideas so the reader can easily follow your argument or the point you are trying to get across.

4. Back it up Have an opinion but back it up - support with data. There are a number of ways you can support your ideas, including explanations, examples, facts, personal experiences, stories, statistics, and quotations. It's best to use a combination of approaches to develop and support your ideas. 5. Separate main ideas Each paragraph should have one main point or idea captured in a topic sentence. The topic sentence is normally the first sentence in the paragraph. Each paragraph should be started by an indentation or by skipping a line. 6. Use bullets or numbers If you are listing or discussing a number of items, use bullets or number your points like I have done in this paper. Here's an example of using bullets. Join the Business Club to:

Increase sales Gain new marketing ideas Make new friends Give back to your profession

7. Write complete sentences A sentence is about someone doing something - taking action. The someone may be a manager, employee, customer, etc. The "doing something - taking action" can include mental processes such as thinking, evaluating, and deciding, or physical

actions such as writing and talking. A good rule to practice is to have subjects closely followed by their verbs. 8. Use short sentences Sentences should be a maximum of 12 to 15 words in length. According to the American Press Institute, sentences with 15 or fewer words are understood 90% of the time. Sentences with eight or fewer words are understood 100% of the time. 9. Be precise and accurate . Words like "large," "small," "as soon as possible," "they," "people," "teamwork," and "customer focus" are vague and imprecise. The reader may interpret these words to mean something different than what you intended. Reduce communication breakdowns by being specific and precise. Define terms as needed. The reader may not understand certain acronyms and abbreviations. 10. Use commas appropriately Use a comma to separate the elements in a series of three or more items:His favorite colors are red, white, and blue. Use a comma to set off introductory elements: After coffee and donuts, the meeting will begin. Use a comma to separate adjectives: That tall, distinguished, goodlooking professor teaches history.

11. Use the correct word Here are several words that cause confusion.

You're is a contraction for "you are" Your means possession, such as "your coat." It's is a contraction for "it is." Its indicates possession. Their means possession/ownership-"their house." There means location. They're is a contraction for "they are."

12. Avoid redundancies It is a redundancy to use multiple words that mean or say the same thing. For example, consider the following:

Redundant - My personal beliefs Beliefs are personal, so just state, beliefs...

My

Redundant - I decided to paint the machine gray in color. Gray a color, so just state, I decided to paint the machine gray.

is

13. Numbers When using numbers in the body of your paper, spell out numbers one through nine, such as "Three men decided" When using numbers 10 or above it's proper to write the number, such as "The report indicated 68 customers"

14. Have a conclusion Would you really enjoy watching a movie or sporting event that had no conclusion? No. The conclusion ties your points together. The reader wants to know the final score - the bottom line message. 15. Edit your work Read what you have written several times.

On your first read, focus on organization and sentence structure. Shorten long sentences. Cross out unnecessary words and phrases. Reorganize material as needed. Read it again and make sure commas are used appropriately and that there is a punctuation mark at the end of every sentence. Read it a third time and focus on word choice. Are there certain words that are vague or unclear? Replace them with specific words. Read what you have written aloud to yourself or to a friend to see if he she (and you) can understand it and improve it in any way.

or

A significant part of good writing involves editing. Very few people can sit down and write a perfect paragraph on their first try. It requires multiple rewrites. Summary You don't have to be a great writer to be successful manager/leader. However you must be able to clearly and succinctly explain your thoughts and ideas in writing. Strive to be simple, clear, and brief. Like any skill, "good writing" requires practice, feedback, and ongoing improvement.

Q.3 How would you prepare yourself for an oral business presentation? ANS: Delivering a formal presentation can be either fairly stress-free or nervewrecking. Your level of comfort can depend on the size of your audience, the critical spectators attending your presentation, or the feedback that you may anticipate. Whatever you may find as a cause for concerns about speaking before a group, never let it be your knowledge about what you will speak. With thorough and effective research about your subject, you will discover that you are already halfway prepared to address your listeners. The following steps can complete your preparation.

1 Study your subject. You may have already been provided great information from which you could pull. But if there are other sources, such as the Internet or experts, use them to enhance your own insight. Doing so can also help you develop more confidence in your speech. 2 In the comfort of your own study lab (wherever that may be), anticipate all types of responding questions from people in your audience: challenging questions, critical questions, crazy questions, and simple questions (the ones which are so simple that you forgot to prepare an answer for). Equip yourself with facts and insight accordingly. For enlightenment on people's views, I have read several message boards and even complaint sites to help me prepare for the unexpected. If your presentation is non-interactive, do not take that for granted. You can still be approached with questions after your speech or after the event where you gave it. 3 Organize the notes from which you will speak. Whether typed or handwritten, you must be able to comprehend them in order to convey them to an audience. So, if you jot your notes down on index cards, write legibly.

Also, bind the notes - paper or cards - that you plan to use during your presentation. Dropping loose papers or cards during your speech should not distract an attentive listener, but it can certainly distract you, the speaker. Make your task a tad bit easier on yourself alleviating the possibility of that problem. 4 If you decide to speak with the aid of a Power Point presentation, bear in mind how you will insert information to be displayed. Don't expect an audience to read lengthy sentences or any paragraph - no matter how much time they are given. By all means, do not prepare yourself to read every word written on that Power Point. The audience does not need to see the back of your head. They don't need to hear your voice drift into a state of monotony, which is what can happen if you read word-for-word from your notes. 5 Practice your presentation in private and be willing to be your own biggest critic. Grab a tape recorder or any recording device to listen to your own speech. 6 As I expressed above, approach the podium with a solid knowledge base about every point your will discuss. 7 Please do not imagine your audience in their underwear. You need to focus. If eye contact with any of those listeners intimidates you, then look just past the last row of people to land your sight on either some empty seats or the wall. There are corners of walls and other inanimate objects where you can place your focus until you find yourself comfortable enough to make brief eye contact with a few friendly or neutral faces. 8 Remember that the last row of listeners need to hear your voice. Unless you have a reliable microphone, be sure to project.

Do not overestimate your listeners' attention spans. Keep their interest. Give your

tone some range (logical range, that is). And wherever your subject and the points from which you speak will allow, engage your audience with illustrations which they can relate to. 10 Please impose neither overly technical terminologies nor acronyms on your audience. While these expressions of intelligence seem effective, they actually reflect a lazy effort to communicate detailed and comprehensible information to listeners. 11 Remember that, at this point, there should be no reason to lose confidence. If you've studied your subject, grasped a clear understanding of it, and followed the tips above, you have the tools to conquer any sharp sensation that you may feel in the pit of your gut going before any group.

Q.5 Distinguish between circulars and notices along with formats? ANS: Notice - A message / information's bringing to all which will be put up in common place Circular- A message / information's bringing to certain group of people belonging to the information's. Like memos, circulars and notices are also written forms of communication within the organization.

The difference between a circular and a notice is that circulars are announcements that are distributed to small or selective groups of people within the organization, whereas notices are meant for a larger group of people. Example - If a manager wants to call a meeting of heads of departments, he will pass around a circular only to the heads, requesting them to attend that meeting. On the other hand, notices generally contain information or announcements that are meant for all the employees of an organization. Example - A list of declared holidays for a calendar year is a notice, since the information is relevant to all employees. A notice is therefore a legal document that has to be put up on an official notice or bulletin board. Let us examine another example of a circular and a notice. Imagine that you are the President of the Student Committee in a management college and wish to hold a meeting to plan for the Annual Management Fest of the college. You will have to send some information to those whom you want to involve in organizing the Fest. You may not want all the students to be involved initially, since it may take a lot of time and there may be too many suggestions. Instead, you may choose to invite only the committee members to discuss details such as the date, venue, duration, how to get sponsors and so on. For this purpose, you may send a circular only to the student committee members, requesting them to attend the meeting. During the meeting, the date and venue may be finalized and various smaller committees may be formed, such as a reception committee, stage committee and so on. You may also decide to get each student to contribute a nominal amount for the Fest

Assignment Set- 2 Q.1 As a part of top management team, how would you communicate to your shareholders about the companys expansion plans? ANS:Commitment 2014 is a firm and confident ambition and it stands for profitable organic growth. Our ambition for 2014 is to become the European benchmark in Universal Customer-focused Banking. We intend to enhance our leadership based on an effective product range and high-quality service, forging close links between retail banking and related specialised businesses. AMBITIOUS TARGETS By 2014, we target ambitious profitability levels underpinned by sound fundamentals: Net banking income of more than 25 billion (20.1 billion in 2010) and net income, Group share of 6 billion to 7 billion (1.3 billion in 2010). Our objective is also to have a cost to income ratio of less than 60% and a return on equity ratio of 10% to 12%. These targets take into account the new Basel III regulatory environment, as it is currently understood. In terms of capital adequacy, Crdit Agricole S.A. will meet the Basel III requirements and Crdit Agricole Group ranking among the strongest banks is confirmed. 2011-2014 STRATEGY Our actions in the period 2011-2014 will be based on three principles. First, we shall stimulate organic growth. To achieve this, we shall enhance growth in retail banking, both in France and elsewhere in Europe; speed up growth in the savings management businesses, one of our undisputed strengths; and finally, focus on growth in investment banking and credit businesses. These businesses will continue to grow, but selectively, because of their heavy capital consumption. The

second principle behind our strategy is that we shall enhance the Group effect by strengthening ties between our business lines. The third principle is that we shall act as a committed and responsible Group in dealing with stakeholders, and in particular our shareholders. OUR SHAREHOLDERS Crdit Agricole S.A. has set up a comprehensive shareholder information and communication framework. The new layout of the Shareholders Club Newsletter, En direct you discover today, is aimed at strengthening this direct link between you and us, and is part of a broader attempt to make full and transparent information available to you. Our dividend policy targets a payout rate of 35% from 2011 (paid in 2012), in cash. Through Commitment 2014, we want to thank our shareholders for their loyalty by paying an increased dividend.

Q.2

ABC Ltd. wants to communicate about its corporate image

to all its stakeholders and also to the general public. As an advisor, how do you recommend them to do it? ANS: Our communications and engagement strategy needs to be simple because we are working in a complex environment - otherwise our actions become confusing to the people we work with. The strategy therefore highlights just 20 key steps we will take to communicate and engage with people effectively. It also explains our simplified approach to outreach work, including events and festivals. Recommendation that: you agree the strategy and its associated annexes.

2 BACKGROUND 2.1 We have significantly improved the way we communicate and engage with people over the last few years. This has been recognised through anecdotal evidence and the recent Customer Service Excellence award. However, we need a strategy to coordinate our activities and guide future business planning. We also need to be able to measure our success more effectively. ABC - our communications and engagement strategy (Annex 1) aims to do this. 3 POLICY CONTEXT 3.1 The adoption of a Communications and Engagement Strategy is action 3.2 in the Business Plan July 2006-March 2009 (revised October 2007). 3.2 The strategy supports the delivery of our Business Plan in its entirety. The messages within it explain that we must be excellent communicators, promoting learning and inspiring people about the National Park in all that we do. 4 OPTIONS 4.1 Option 1: You agree the recommendation Option 2: You agree the recommendation with further development of some steps Option 3: You do not agree the recommendation 5 PROPOSALS 5.1 The Communications and Engagement Task and Finish Group was established to develop this strategy. It includes staff from the Communications and Learning Team and members. They undertook a SWOT analysis (Strengths, Weaknesses, Opportunities and Threats) and agreed the simple approach recommended. They commented on the 20 key steps ensuring that they were strategic and not too

detailed. Option 1 is recommended for approval and it is hoped that members will be involved in the delivery and review of the strategy as it progresses. 6 BEST VALUE IMPLICATIONS 6.1 The statutory duty of best value requires organisations to consult service users and other stakeholders about services and priorities. The duty was revised in the Local Government and Public Involvement in Health Bill in May 2007 to engage more with hard-to-reach groups and to secure participation of communities in delivery of local public services. This strategy supports the delivery of this function by focusing on engagement with partners and local people, particularly through two of our four key Lake District National Park Authority Agenda Item: 9 Authority: 22 May 2008 requirements for effective communications - listening and interpreting demands; unraveling complex issues. 7 FINANCE CONSIDERATIONS 7.1 There are financial implications in the delivery of the strategy. You have already agreed revenue growth bids (totalling 45,000) to support step 9 in the strategy and development of Level One events and festivals which we sponsor (Annex B). You have also agreed existing budgets to support ongoing work led by the Communications and Learning Team. Agreement of any additional budgetary requirements to deliver all steps in the strategy will form part of the corporate planning and performance cycle.

8 RISK 8.1 Each step in the strategy has a different degree of risk which will be managed through respective service plans. Overall, if we do not develop our communications and engagement activities, there is a significant risk that we lose our Customer Service Excellence standard. 9 LEGAL CONSIDERATIONS 9.1 Effective communication is key to performance of our statutory objectives. The proposed strategy will assist with good governance by improving communication within the Authority and will help clarify relationships with partners. 10 HUMAN RESOURCES 10.1 The Communications and Learning Team, including its Outreach Unit, are part of the established structure and will take a clear coordinating role in delivering the strategy. The strategy also makes it clear that new approaches to communications and engagement should be a part of existing roles across the organisation. This will be dealt with through Service Planning and Personal Development Reviews. 11 DIVERSITY IMPLICATIONS 11.1 This strategy is of high importance to diversity. In the past, our approach to targeting groups may have had an adverse impact on particular groups already using our services. The strategy explains that we will now base our actions on the needs of different groups and clear evidence. Annex A (Reaching Out) explains how we will not target people, but work with focus groups to identify needs over time. We want toreach out to everyone and use the National Park to build links between different cultures.

12 SUSTAINABILITY 12.1 The strategy will contribute to the promotion of learning and inspiring people about all aspects of sustainability which is at the heart of the Vision for the National Park. Author/Post Bob Sutcliffe, Head of Plans and Communications Date Written 2 May 2008 Q.3 What is oral business communication? Explain its

benefits to the organisation and to the individual employee. ANS: According to a 2005 study published in the Journal of Employment Counseling, oral communication skills are being increasingly sought after by employers. When surveying over 100 successful businesses, researchers found that more and more employers are emphasizing the development of good speaking skills in their employees. With this in mind, the concept of oral communication is an important idea to study and understand in the context of business. Presentations: One form of oral communication in a business setting is a presentation. Presentations are usually an organized conveyance of information to a group of people. Stylistically, they tend to be far more formal than informal, and rely more heavily on data and facts than they do analysis. Presentations are sometimes more persuasive in nature, like a pitch for an ad campaign, but tend to be informative more often, such as an employee briefing or a report on quarterly earnings. Presentations may include some dialog after the sender of the message has finished their speech, but they are, by and large, much more monologue reliant. This makes

it important for the speaker to anticipate possible objections to the message and address them in the actual speech. Client Interaction: Another form of oral communication in business encompasses interaction with clients. Depending on the level of connection between the employee and the client, the communication in these interactions can range from incredibly formal to informal and casual. These interactions usually include a combination of data and analysis, and will be more persuasive than informative in nature, as the employee is trying to encourage continued and expanded business with the client. Because of the nature of these interactions, the communication is definitely a dialog, making listening skills incredibly important. Interoffice Interaction: Oral communication in the office can be referred to as interoffice interaction. This is comprised of conversations with superiors, subordinates and co-workers. Depending on the levels of power separation between the individuals engaging in conversation, the communication will fluctuate between formal and informal, though it should always remain professional. Conversations in this context may reference data, but will be much more analysis heavy, and will be a dialog by nature. Benefits: Oral communication in business provides a variety of benefits. First, oral communication is accompanied by nonverbal signifiers, which provides context

that can enhance understanding in the communication process. Posture, facial expressions, and habitual movements may provide clues as to an individuals feelings about the ideas being discussed. Even in telephone conversations, pitch, rate, volume and tone of the respective speakers can help in understanding sentiments. Oral communication also provides a springboard for relational development. Unlike with email, memos and chat functions, which tend to take a task-oriented approach to communication, the immediacy involved in oral communication allows for instant feedback and a more relational approach. This is important, as strong relationships in business often lead to more profitable and productive cooperation. Q.4. Give short notes on communication network in the organisation? ANS: Networks are another aspect of direction and flow of communication. Bavelas has shown that communication patterns, or networks, influence groups in several important ways. Communication networks may affect the group's completion of the assigned task on time, the position of the de facto leader in the group, or they may affect the group members' satisfaction from occupying certain positions in the network. Although these findings are based on laboratory experiments, they have important implications for the dynamics of communication in formal organizations. There are several patterns of communication: "Chain", "Wheel", "Star",

"All-Channel" network, "Circle". The Chain can readily be seen to represent the hierarchical pattern that characterizes strictly formal information flow, "from the top down," in military and some types of business organizations. The Wheel can be compared with a typical autocratic organization, meaning oneman rule and limited employee participation. The Star is similar to the basic formal structure of many organizations. The AllChannel network, which is an elaboration of Bavelas's Circle used by Guetzkow, is analogous to the free-flow of communication in a group that encourages all of its members to become involved in group decision processes. The All-Channel network may also be compared to some of the informal communication networks. If it's assumed that messages may move in both directions between stations in the networks, it is easy to see that some individuals occupy key positions with regard to the number of messages they handle and the degree to which they exercise control over the flow of information. For example, the person represented by the central dot in the "Star" handles all messages in thegroup. In contrast, individuals who occupy stations at the edges of the pattern handle fewer messages and have little or no control over the flow of information.These "peripheral" individuals can communicate with only one or two other persons and must depend entirely on others to relay their messages if they wish to extend their range. In reporting the results of experiments involving the Circle, Wheel, and Star configurations, Bavelas came to the following tentative conclusions. In patterns with positions located centrally, such as the Wheel and the Star, an organization quickly develops around the people occupying these central positions. In such

patterns, the organization is more stable and errors in performance are lower than in patterns having a lower degree of centrality, such as the Circle. However, he also found that the morale of members in high centrality patterns is relatively low. Bavelas speculated that this lower morale could, in the long run, lower the accuracy and speed of such networks. In problem solving requiring the pooling of data and judgments, or "insight," Bavelas suggested that the ability to evaluate partial results, to look at alternatives, and to restructure problems fell off rapidly when one person was able to assume a more central (that is, more controlling) position in the information flow. For example, insight into a problem requiring change would be less in the Wheel and the Star than in the Circle or the Chain because of the "bottlenecking" effect of data control by central members. It may be concluded from these laboratory results that the structure of communications within an organization will have a significant influence on the accuracy of decisions, the speed with which they can be reached, and the satisfaction of the people involved. Consequently, in networks in which the responsibility for initiating and passing along messages is shared more evenly among the members, the better the group's morale in the long run.

Q. 5 What are the different types of business letters? Explain with example. ANS: Business letter is an old form of official correspondence. A business letter is written by an individual to an organization or an organization to another organization. Business letters are written for various purposes. One writes a letter to enquire information, apply for a job, acknowledge someone's work, and appreciate one's job done, etc. As the motive of writing the letter is different, the

style of the letter changes and you get different types of business letters. The various types of business letters are used by different people to serve their purpose of sending the message across. Let's take look at the most common types of business letters: Acknowledgement Letter : This type of letter is written when you want to acknowledge some one for his help or support when you were in trouble. The letter can be used to just say thanks for something you have received from some one, which is of great help to you. Apology Letter : An apology letter is written for a failure in delivering the desired results. If the person has taken up a task and he fails to meet the target then he apologizes and asks for an opportunity to improve in this type of letter. Appreciation Letter : An appreciation letter is written to appreciate some one's work in the organization. This type of letter is written by a superior to his junior. An organization can also write an appreciation letter to other organization, thanking the client for doing business with them. Complaint Letter : A complaint letter is written to show one that an error has occurred and that needs to be corrected as soon as possible. The letter can be used as a document that was used for warning the reader. Inquiry Letter : The letter of inquiry is written to inquire about a product or service. If you have ordered a product and yet not received it then you can write a letter to inquire when you will be receiving it.

Order Letter : This letter is as the name suggests is used for ordering products. This letter can be used as a legal document to show the transaction between the customer and vendor. Letter of Recommendation : This type of letter is written to recommend a person for a job position. The letter states the positive aspects of the applicant's personality and how he/she would be an asset for the organization. Letter of recommendation is even used for promoting a person in the organization.

Assignment Set- 1 Master Administration MBA Semester -1 MB0040 - STATISTICS FOR MANAGEMENT Q. 1 What is the difference between a qualitative and quantitative variable? ANS: Scientific experiments will normally have three types of variables; controlled, independent and dependent. Variables are a condition or factor that is used in testing a hypothesis and generating a conclusion. These three types of variables can also be quantitative or qualitative in nature. Qualitative: By definition something that is qualitative concerns or describes a quality. A qualitative variable is a descriptive. Qualitative variable are sometimes referred to as categorical. The variable may be colors in the light spectrum or a comparison between red and green grapes. Qualitative variables can influence the outcome of an experiment or research because they can influence other factors or parameters. Qualitative variables are frequently used in social research. Qualitative research is considered to be inductive. Quantitative: By definition something that is quantitative can be expressed as a quantity or number. Quantitative variables are something that can be measured. Quantitative variables are numerical. A quantitative variable can be a percentage of something, a number of units or any other measurement. of Business -

Temperature is a quantitative value or variable by the number of degrees. Speed, area population, voltage and time are all examples of quantitative variables that can be measured. Quantitative variables are most often considered to be deductive in nature. Deduction and induction in experimentation and research: Deduction works from a general idea to a specific idea. Deductive research starts with a theory, forms a hypothesis, gathers observations and then confirms or disproves the original thought. Induction works in the reverse. Inductive experimentation will start with an observation and then look for patterns in the observation. Once patterns form a hypothesis is developed. The hypothesis is then tested for a resulting theory. The best results in experimentation come from having only one independent variable. The controlled variable is something that does not change and must remain constant. The independent variable is the variable that is changed by the researcher. The dependent value is the variable that changes due to the independent variable. An example of quantitative variables in an experiment would be testing the change in speed on a turntable as additional weight is applied. The turntable itself is the controlled variable. The experimenter will only use one. The independent quantitative variable is the amount of weight applied for each measurement. The dependent quantitative variable is the resulting speed that is measured. An example of a qualitative variable in testing would be the drying time require for red and green grapes at a constant temperature. The outcome, or dependent

variable, of time is measured and therefore quantitative. The controlled variable being used is temperature, also quantitative. The independent variable is qualitative, the difference between red and green grapes. In this particular example the weight of each grape, a quantitative variable would also need to be consistent or controlled.

Q 2. a) Explain the steps involved in planning of a statistical survey? ANS: I've explained the fundamentals of creating and executing surveys. We've explored how to construct questions so they yield accurate, useful data. We've discussed how to boost your response rate, thereby squeezing more value from your efforts. And we've talked briefly about the various strategies you can employ to solicit participation from your respondents. But, everything begins with the initial plan; you shouldn't approach it haphazardly. In this article, I'll describe the 5 steps that you and your design team should follow to plan a successful survey execution. Step #1: Identify Your Objective This first step is more involved than you might think and is the foundation upon which the other steps depend. It's not enough to say, "I need to know if customers are satisfied" or "I want to measure employee morale." You should know in advance the precise reasons you want to collect data, what you hope to learn from the information, and the actions you intend to take once you and your team have reviewed the responses. Unless you have already identified a specific reason for conducting your survey, including how it will add value to your business or organization, you shouldn't move forward.

Step #2: Determine How Much It Will Cost Design and deployment can be expensive depending upon its scope, the information you hope to collect, and your population size. Some of the costs aren't immediately obvious. For example, if you're using employees to approach participants, you'll need to pay for their time. If you're conducting a survey by mail, you'll have to pay postage. Analyzing the data also carries a cost. Plan conservatively to ensure you have the budget to see the project through its completion. Step #3: Plan The Logistics You'll need to choose your participant pool, figure out how to approach them, and determine how much time your project will require. Don't take any of these items lightly. Your selection of the people you choose to approach will depend upon the type of information you hope to collect. For example, surveying single men between ages 30 and 39 will yield different data than married couples over 60. Your strategy for approaching them can vary between phone, email, in person interviews, postal mail, etc. The time you'll need to complete your survey will be based largely upon these factors. Step #4: Define Your Resources Beyond your budget, time constraints, and other limitations; you'll need to identify resources (both internal and external) upon which you'll rely for expertise and assistance. For example, if you're measuring employee satisfaction, you'll want to enlist the help of your organization's human resources department. If you're

outsourcing the design of your survey, you'll need to identify and interview a number of experienced agencies. During this step, you should also determine the internal personnel who will need to see your team's analysis of the data (i.e. president, CFO, accounting department, etc.). Step #5: Map The Steps To Completion Before you execute your survey, take the time to create a chronological timetable of the project. It should detail each step in the process, including who is responsible for collecting the information needed to complete each step and the specific tasks that are involved along the way. Without creating this plan ahead of time, you risk wasting finite resources as a result of confusion and lack of accountability. Successful Deployment Creating and executing effective surveys that yield useful data require a lot of time and upfront planning. If you approach the task methodically and follow the 5 steps we've covered above, you'll dramatically improve the likelihood of success. Remember your objective: to collect accurate information that you can take action upon for a predefined benefit to your organization. In an upcoming article, we'll explore the deployment in more detail. b) What are the merits & Demerits of Direct personal

observation and Indirect Oral Interview? ANS: In the direct personal observation method, the investigator collects data by having direct contact with units of investigation.The accuracy of data depends

upon the ability, training and attitude of the investigator. The direct personal observation method is suitable where - The scope of investigation is narrow - Investigation is confidential and requires personal attention of the investigator - Accuracy of data is important Merits and demerits of direct personal observation Merits: 1) We get original data. 2) we get more accurate and reliable data. 3) Satisfactory information can be extracted by the investigator through indirect questions. 4) Data are homogeneous and comparable. 5) Additional information can be gathered. 6) Misinterpretation of questions can be avoided. Demerits: 1) It is time consuming and costs more. Indirect oral interview is used when area to be covered is large. The data is collected from a third party or witness or head of institution. This method is generally used by police department.

Merits and demerits of indirect oral interview Merits: 1) It is economical in terms of time, cost and manpower. 2) Confidential information can be collected. 3) Information is likely to be unbiased and reliable.

Demerits: 1) The degree of accuracy of information is less. Q 3. a) What is the main difference between correlation analysis and regression analysis? ANS: (1) The correlation answers the STRENGTH of linear association between paired variables, say X and Y. On the other hand, the regression tells us the FORM of linear association that best predicts Y from the values of X. (2a) Correlation is calculated whenever: * both X and Y is measured in each subject and quantify how much they are linearly associated. * in particular the Pearson's product moment correlation coefficient is used when the assumption of both X and Y are sampled from normally-distributed populations are satisfied * or the Spearman's moment order correlation coefficient is used if the assumption of normality is not satisfied. * correlation is not used when the variables are manipulated, for example, in experiments.

(2b) Linear regression is used whenever: * at least one of the independent variables (Xi's) is to predict the dependent variable Y. Note: Some of the Xi's are dummy variables, i.e. Xi = 0 or 1, which are used to code some nominal variables. * if one manipulates the X variable, e.g. in an experiment. (3) Linear regression are not symmetric in terms of X and Y. That is interchanging X and Y will give a different regression model (i.e. X in terms of Y) against the original Y in terms of X. On the other hand, if you interchange variables X and Y in the calculation of correlation coefficient you will get the same value of this correlation coefficient. (4) The "best" linear regression model is obtained by selecting the variables (X's) with at least strong correlation to Y, i.e. >= 0.80 or <= -0.80 (5) The same underlying distribution is assumed for all variables in linear regression. Thus, linear regression will underestimate the correlation of the independent and dependent when they (X's and Y) come from different underlying distributions. b) In a multiple regression model with 12 independent variables, what are the degrees of freedom for error? Explain? ANS: Consider a random sample of n observations (xi1, xi2, . . . . , xip, yi), i = 1, 2, . . . , n. The p + 1 random variables are assumed to satisfy the linear model yi
0 1

xi1

2xi2

pxip

+ ui i = 1, 2, . . . , n

where ui are values of an unobserved error term, u, and. the unknown parameters are constants. Assumptions

The error terms ui are mutually independent and identically distributed, with mean = 0 and constant variances E [ui] = 0 V [ui] =

This is so, because the observations y1, y2, . . . ,yn are a random sample, they are mutually independent and hence the error terms are also mutually independent The distribution of the error term is independent of the joint distribution of x
i,

x 2, . . . , x p
0 1 2 p

are constants.

Equations relating the n observations can be written as:

can be estimated using the least squares

procedure, which minimizes the sum of squares of errors.

Minimizing the sum of squares leads to the following equations, from which the

Geometrical Representation The problem of multiple regression can be geometrically represented as follows. We can visualize that n observations (xi1, xi2, ..xip, yi) i = 1, 2, .n are represented as points in a (p+1) - dimensional space. The regression problem is to determine the possible hyper-planes in the p - dimensional space, which will be the best- fit. We use the least squares criterion and locate the hyper-plane that minimizes the sum of squares of the errors, i.e., the distances from the points around the plane (observations) and the point on the plane. (i.e. the estimate ). = a+b1x1+b2x2++bpxp Standard error of the estimate

Se = where yi = the sample value of the dependent variable i = corresponding value estimated from the regression equation

n = number observations p = number of predictors or independent variable The denominator of the equation indicates that in multiple regression with p independent variables, the standard error has n-p-1 degrees of freedom. This happens because the degrees of freedom are reduced from n by p+1 numerical constants a, b1, b2, ..bp, that have been estimated from the sample. Fit of the regression model The fit of the multiple regression model can be assessed by the Coefficient of Multiple determination, which is a fraction that represents the proportion of total variation of y that is explained by the regression plane.

Sum of squares due to error

SSE = Sum of squares due to regression

SSR = Total sum of squares

SST = Obviously, SST = SSR + SSE The ratio SSR/SST represents the proportion of the total variation in y explained by the regression model. This ratio, denoted by R2, is called the coefficient of multiple determination. R2 is sensitive to the magnitudes of n and p in small samples. If p is large relative to n, the model tends to fit the data very well. In the extreme case, if n = p+1, the model would exactly fit the data. A better goodness of fit measure is the adjusted R2, which is computed as follows:

Adjusted R2= 1 - (

) (1-R2)

=1-

Statistical inferences for the model The overall goodness of fit of the regression model (i.e. whether the regression model is at all helpful in predicting the values of y can be evaluated, using an Ftest in the format of analysis of variance. Under the null hypothesis: Ho: 1 = 2 = ... = p = 0, the statistic

= has an F-distribution with p and n--1 degrees of freedom ANOVA Table for Multiple Regression Source of Variation Sum of Squares Degrees Mean of freedom Regression Error Total SSR SSE SST p (n-p-1) (n-1) MSR MSE MSR/MSE Squares F ratio

Whether a particular variable contributes significantly to the regression equation can be tested as follows: For any specific variable xi, we can test the null hypothesis Ho: i = 0, by computing the statistic

t=

and performing a one or two tailed t-test with n-p-1 degrees of freedom. Standardized regression coefficients The magnitude of the regression coefficients depends upon the scales of measurement used for the dependent variable y and the explanatory variables included in the regression equation. Unstandardized regression coefficients cannot be compared directly because of differing units of measurements and different variances of the x variables. It is therefore necessary to standardize the variables for meaningful comparisons. The estimated model i = bo+b1xi1+b2xi2+.bpxip can be written as:

+ The expressions in the parentheses are standardized variables; bs; are unstandardized regression coefficients and s1, s2, sp are the standard deviations of variables x1, x2, .xp and sx is the standard deviation of variable y. The coefficients (bisi)/sy, j=1,2,,p are called standardized regression coefficients. The standardized regression coefficient measures the impact of a unit change in the standardized value of xi on the standardized value of y. The larger the magnitude of standardized bi, the more xi contributes to the prediction of y. However, the regression equation itself should be reported in terms of the unstandardized regression coefficients so that prediction of y can be made directly from the x variables.

Multiple Correlation Multiple correlation coefficient, R, is a measure of the strength of the linear relationship between y and the set of variables x1, x2, xp. It is the highest possible simple correlation between y and any linear combination of x1,x2,.,xp. This property explains that the computed value of R is never negative. In this sense, the least squares regression plane maximizes the correlation between the x variables and the dependent variable y. Hence, it represents a measure of how well the regression equation fits the data. When the value of the multiple correlation R is close to zero, the regression equation barely predicts y better than sheer chance. A value of R close to 1 indicates a very good fit. Partial Correlation A useful approach to study the relationship between two variables x and y in the presence of a third variable z is to determine the correlation between x and y after controlling the effect of z. This correlation is called partial correlation. Partial correlation is the correlation of two variables while controlling for a third or more other variables. For example r12.34 is the correlation of variables 1 and 2, controlling for variables 3 and 4. If partial correlation r12.34 is equal to uncontrolled correlation r12 , it implies that the control variables have no effect on the relationship between variables 1 and 2.. If partial correlation is nearly equal to zero, it implies that the correlation between original variable is spurious. Partial correlation coefficient is a measure of the linear association between two variables after adjusting for the linear effect of a group of other variables. If the number of other variables is equal to 1, the partial correlation coefficient is called

the first order coefficient. If the number of other variables is equal to 2, the partial correlation coefficient is called the second order coefficient, and so on. First order Partial Correlation The first order partial correlation between xi and xj holding constant xl is computed by the following formula

rij.l = where rij, ril and rjl are zero order (Pearsons r coefficient) Second order Partial Correlation Correlation between xi and xj holding constant xl and xm is computed by the following formula:

rij.lm = where rij, rim.l, rjm.l are first order partial correlation coefficients. Statistical significance of partial correlation coefficients can be tested by using a test statistic similar to the one for simple correlation coefficient.

t= where q is the number of variables held constant. The value of t is compared with tabulated t for n-q-2 degrees of freedom.

Multicollinearity In practice, the problem of multicollinearity occurs when some of the x variables are highly correlated. Multicollinearity can have significant impact on the quality and stability of the fitted regression model. A common approach to multicollinearity problem is to omit explanatory variables. For example if x1 and x2 are highly correlated (say correlation is greater than 0.9), then the simplest approach would be to use only one of them, since one variable conveys essentially all the information in the other variable. The simplest method for detecting multicollinearity is the correlation matrix, which can be used to detect if there are large correlations between pairs of explanatory variables. When more subtle patterns of correlation coefficients exist, the determinant of the correlation matrix computed by IDAMS can be used to detect multicollinearity. The determinant of the correlation matrix represents as a single number the generalized variance in the set of predictor variables, and varies from 0 to 1. The value of the determinant near zero indicates that some or all explanatory variables are highly correlated. The value of the determinant equal to zero indicates a singular matrix, which indicates that at least one of the predictors is a linear function of one or more other predictors. Another approach is to compute the tolerance associated with a predictor. The tolerance of xi is defined as 1 minus the squared multiple correlation between that xi and the remaining x variables. When tolerance is small, say less than 0.01, then it would be expedient to discard the variable with the smallest tolerance. The inverse of the tolerance is called the variance inflation factor (VIF).

Stepwise Regression Stepwise regression is a sequential process for fitting the least squares model, where at each step a single explanatory variable is either added to or removed from the model in the next fit. The most commonly used criterion for the addition or deletion of variables in stepwise regression is based on partial F-statistic:

= The suffix Full refers to the larger model with p explanatory variables, whereas the suffix Reduced refers to the reduced model with ( variables. Forward selection Forward selection procedure begins with no explanatory variable in the model and sequentially adds a variable according to the criterion of partial F- statistic. At each step, a variable is added, whose partial F- statistic yields the smallest p - value. Variables are entered as long as the partial F-statistic p-value remains below a specific maximum value (PIN). The procedure stops when the addition of any of the remaining variables yields a partial p-value > PIN. This procedure has two limitations. Some of the variables never get into the model and hence their importance is never determined. Another limitation is that a variable once included in the model remains there throughout the process, even if it loses its stated significance, after the inclusion of other variable(s). ) explanatory

Backward elimination The backward elimination procedure begins with all the variables in the model and proceeds by eliminating the least useful variable at a time. A variable, whose partial F p-value is greater than a prescribed value, POUT, is the least useful variable and is therefore removed from the regression model. The process continues, until no variable can be removed according to the elimination criterion. Stepwise procedure The stepwise procedure is a modified forward selection method which later in the process permits the elimination of variables that become statistically nonsignificant. At each step of the process, the p-values are computed for all variables in the model. If the largest of these p-values > POUT, then that variable is eliminated. After the included variables have been examined for exclusion, the excluded variables are re-examined for inclusion. At each step of the process, there can be at the most one exclusion, followed by one inclusion. It is necessary that PIN POUT to avoid infinite cycling of the process. Regression with Qualitative Explanatory Variables Sometimes, explanatory variables for inclusion in a regression model are not interval scale; they may be nominal or ordinal variables. Such variables can be used in the regression model by creating dummy (or indicator) variables. Dichotomous Variables Dichotomous variables do not cause the regression variables to lose any of their properties. Since they have two categories, they manage to trick least squares,

while entering into the regression equation as interval scale variables with just two categories. Consider for example, the relationship between income and gender y = a + bx where y = income of an individual, and x = a dichotomous variable, coded as 0 if female 1 if otherwise The estimated value of y is =a if x = 0 =a+b if x = 1 Since our best estimate for a given sample is the sample mean, a is estimated as the average income for females and a+b is estimated as average income for males. The regression coefficient b is therefore
male

female

In effect, females are considered as the reference group and males income is measured by how much it differs from females income.

Polytomous Variables Consider, for example, the relationship between the time spent by an academic scientist on teaching and his rank. y = a+bx where y is the percentage of work time spent on teaching x is a polytomous variable rank with three modalities: 1 = Professor 2 = Reader 3 = Lecturer We create two dummy variables: X1 = 1 if rank = Professor 0 if otherwise X2 = 1 if rank = Reader 0 if otherwise Note that we have created two dummy variables to represent a trichotomous variable. If we create a third dummy variable X3 (score 1; if rank = Lecturer, and 0 otherwise), the parameters of the regression equation cannot be estimated uniquely. This is because if the score of any respondent on X1 and X2 is known, it would

always be possible to predict his score on X3. For example if a respondent has score 0 on X1 (not Professor) and 0 on X2 (not Reader), then the respondent is certainly a Lecturer (i.e., score 1 on X3). This represents a situation of perfect multicollinearity. Hence the general rule for creating dummy variables is: Number of dummy variables = Number of modalities minus 1. Statistical significance of regression coefficients and Multiple R2 is determined in the same way as for interval scale explanatory variables.

Q 5. a) Discuss what is meant by Quality control and quality improvement? ANS: Quality is a much more complicated term than it appears. Dictionary definitions are usually inadequate in helping a quality professional understand the concept. It seems that every quality expert defines quality is a somewhat different way. There are a variety of perspectives that can be taken in defining quality (e.g. customer's perspective, specification-based perspective). Are there commonalities among these definitions? Is any one definition "more correct" than the others? Is one quality expert "right" and the others "wrong"? Quality professionals constantly debate this question. The editors of Quality Digest say that defining the word "quality" is "no simple endeavor." They asked, in their December 1999 issue, for readers to send them their definitions of quality to be gathered and posted on Quality Digest Online. A modern definition of quality derives from Juran's "fitness for intended use." This definition basically says that quality is "meeting or exceeding customer expectations." Deming states that the customer's definition of quality is the only one that matters.

Quality assurance (QA) is a broad concept that focuses on the entire quality system including suppliers and ultimate consumers of the product or service. It includes all activities designed to produce products and services of appropriate quality. According to ASQ, QA includes all those planned or systematic actions necessary to provide adequate confidence that a product or service will satisfy given needs. [Source: ASQ Statistics Division, Glossary & Tables for Statistical Quality Control, 1983]. Quality control (QC) has a narrower focus than quality assurance. Quality control focuses on the process of producing the product or service with the intent of eliminating problems that might result in defects. According to ASQ, QC includes the operational techniques and the activities which sustain a quality of product or service that will satisfy given needs; also the use of such techniques and activities. [Source: ASQ Statistics Division, Glossary & Tables for Statistical Quality Control, 1983]. Quality management is the totality of functions involved in the determination and achievement of quality (includes quality assurance and quality control). [Source: ASQ Statistics Division, Glossary & Tables for Statistical Quality Control, 1983]. External customers usually come to mind first. These are the people outside our organization who receive our goods and services. But even here there is some confusion. If we sell our products to a wholesaler, is he our only customer? How about the retailer and the ultimate consumer?

Internal customers are often forgotten or taken for granted. In an assembly line operation, the next station downstream from ours is an internal customer for our work. The Purchasing Dept. who receives a control report from the Accounting Dept. is the Accounting Dept's. internal customer. Second grade teachers are internal customers for first grade teachers. Now, let's discuss meeting or exceeding customer expectations. Meeting customer expectations results in a satisfied customer. But where is the competitive advantage in that? Have you eaten in a restaurant in the past month? If so, did you select a restaurant that you expected would dissatisfy you? Probably not. You selected from a list of restaurants that you expected would satisfy you. So, satisfying customers merely keeps you in the game. Delighting customers (exceeding customer expectations) is where competitive advantage can be found. Restaurants that deliver larger than expected portions or lower than expected prices or better than expected service or better than expected ambiance (order winners) have a competitive advantage over restaurants that simply satisfy customers (possess only order qualifiers). A number of scholars in the quality field have developed lists of dimensions that define quality for a product and/or a service. David Garvin developed a list of 8 dimensions of product quality. Evans and Lindsay provide a list of 8 dimensions of service quality. These are general lists and serve as good starting points. But, current research indicates that in terms of service quality, the dimensions are different for different industries. So Evans and Lindsay's list may not apply equally well to, for example, health care services and food services. Parasuraman, et. al. developed a general list of 5 service dimensions that they tested in 4 types of service industry, but the applicability of these dimensions in other industries is unknown.

Developing a list of quality dimensions for a specific service industry requires determining what is important to customers. Methodologies which are appropriate for this would include focus groups and surveys. The quality dimensions for hospitals (KQCAH Scale) that are shown in the Lesson 1 Presentation were developed using focus groups conducted with recently discharged patients and their families, and with hospital personnel. Knowledge of these dimensions facilitates the measurement of patient satisfaction by hospitals. Hospitals know that they are measuring dimensions that are important to patients. b) What are the limitations of a quality control charts? ANS: The limitations of a quality control charts: The quality control chart is based on the research of Villefredo Pareto. He found that approximately 80 percent of all wealth of Italian cities he researched was held by only 20 percent of the families. The Pareto principle has been found to apply in other areas, from economics to quality control. Pareto charts have several disadvantages, however. Easy to Make but Difficult to Troubleshoot percent of issues that cause the majority of problems in order to have the greatest impact. However, one of the disadvantages of Pareto charts is that they provide no insight on the root causes. For example, a Pareto chart will demonstrate that half of all problems occur in shipping and receiving. Failure Modes Effect Analysis, Statistical Process Control charts, run charts and cause-and-effect charts are needed to determine the most basic reasons that the major issues identified by the Pareto chart are occurring.

Multiple Pareto Charts May Be Needed Pareto charts can show where the major problems are occurring. 6. a) Suggest a more suitable average in each of the following cases: (i) Average size of ready-made garments. ANS: (i) Average size of ready-made garments: Arithmetic mean will be used because it is continuous and additive in nature. (ii) Average marks of a student. ANS: Average marks of a student. : Arithmetic mean will be used because it is the data are in the interval and the distribution is symmetrical. b) State the nature of symmetry in the following cases: (i) When median is greater than mean? ANS: (i) When median is greater than mean : the data are "skewed to the left", with a long tail of low scores pulling the mean down more than the median.

Assignment Set- 2 Q 1. What are the characteristics of a good measure of central tendency? ANS: Characteristics of a Good Average (i) It should be rigidly defined. If an average is left to the estimation of an observer and if it is not a definite and fixed value it cannot be representative of a series. The bias of the investigator in such cases would considerably affect the value of the average. If the average is rigidly defined; this instability in its value would be no more, and it would always be a definite figure, (ii) It should be based on all the observations of the series. If some of the items of the series are not taken into account in its Calculation the average cannot be said to be a representative one. As we shall see later on there are some averages which do not take into account all the values of a group and to this extent they are not satisfactory averages. (iii) It should be capable of further algebraic treatment. If an average dose not possess this quality, its use is bound to be very limited. It will not be possible to calculate, say, the combined average of two or more series from their individual averages; further it will not be possible to study the average relationship of various parts of a variable if it is expressed as the sum of two or more variables. Many other similar studies would not be possible if the average is not capable of further algebraic treatment. (iv) It should be easy to calculate and simple to follow. If the calculation of the average involves tedious mathematical processes it will not be readily understood and its use will be confined only to a limited number of persons. It can never be a

popular average. As such, one of the qualities of a good average is that it should not be too abstract or mathematical and there should be no difficulty in its calculation. Further, the properties of the average should be such that they can be easily understood by persons of ordinary intelligence. (v) It should not be affected by fluctuations of sampling. If two independent sample studies are made in any particular field, the averages thus obtained, should not materially differ from each other. No doubt, when two separate enquires are made, there is bound to be a difference, in the average values calculated but in some cases this difference would be great while in others comparatively less. These averages in which this difference, which is technically called "fluctuation of sampling" is less, are considered better than those in which its difference is more.

One more thing to be remembered about averages is that the items whose average is being calculated should form a homogenous group. It is absurd to talk about the average of a man's height and his weight. If the data from which an average is being calculated are not homogeneous, misleading conclusions are likely to be drawn. To find out the average production of cotton cloth per mill, if big and small mills are not separated the average would be unrepresentative. Similarly, to study wage level in cotton mill industry of India, separate averages should be calculated for the male and female workers. Again, adult workers should be separately studied from the juvenile group. Thus we see that as far as possible, the data from which an average is calculated should be a homogeneous lot. Homogeneity can be achieved either by selecting only like items or by dividing the heterogeneous data into a number of homogeneous groups.

(b) What are the uses of averages? ANS: The use or application of a particular average depends upon the purpose of the investigation. Some of the cases of different averages are as follows: Arithmetic Mean Arithmetic mean is considered an deal average. It is frequently used in all the aspects of life. It possesses many mathematical properties and due to this it is of immense utility in further statistical analysis. In economic analysis arithmetic mean is used extensively to calculate average production, average wage, average cost, per capital income exports, imports, consumption, prices, etc. When different items of a series have different relative importance, then weighted arithmetic mean is used. Geometric Mean Use of Geometric mean is important in a series having items of wide dispersion. It is used in the construction of index number. The averages of proportions, percentages and compound rates are computed by geometric mean. The growth of population is measured in it as population increases in geometric progression. Harmonic Mean Harmonic mean is applied in the problems where small items must get more relative importance than the large ones. It is useful in cases where time, speed, values given in quantities, rate and prices are involved. But in practice, it has little applicability. Median and partition Values

Median and partition values are positional measures of central tendency. There are mainly used in the qualitative cases like honestly, intelligence, ability, etc. In the distributions which are positively skewed, median is a more suitable average. These are also suitable for the problems of distribution of income, wealth, investment, etc. Mode Mode is also positional average. Its applicability of daily problems is increasing. Mode is used to calculate the 'modal size of a collar', 'modal size of shore', or 'modal size of ready-made garments' etc. It is also used in the sciences of Biology, Meteorology, Business and Industry. Q 3. What is test statistic? Why do we have to know the distribution of a test statistic? ANS: A statistic is calculated from the sample. To begin with we assume that the hypothesis about the population parameter is true. We compare the value of the statistic with the hypothetical value of the parameter. If the difference between them is small, the hypothesis is accepted and if the difference between them is large, the hypothesis is rejected. A statistic on which the decision can be based whether to accept or reject a hypothesis is called test statistic. It is important to remember that a test-statistic does not prove the hypothesis to be correct but if furnishes as evidence against the hypothesis. Some of the test statistics to be discussed later are Z, t and Chi - Square. Statistics is a diverse subject and thus the mathematics that are required depend on the kind of statistics we are studying. A strong background in linear algebra is needed for most multivariate statistics, but is not necessary for introductory statistics. A background in Calculus is useful no matter what branch of statistics is

being studied, but is not required for most introductory statistics classes.At a bare minimum the student should have a grasp of basic concepts taught in Algebra and be comfortable with "moving things around" and solving for an unknown. Refresher Course Most of the statistics here will derive from a few basic things that the reader should become acquainted with. Absolute Value

If the number is zero or positive, then the absolute value of the number is simply the same number. If the number is negative, then take away the negative sign to get the absolute value. Examples

|42| = 42 |-5| = 5 |2.21| = 2.21

Factorials A factorial is a calculation that gets used a lot in probability. It is defined only for integers greater-than-or-equal-to zero as:

Examples In short, this means that: 0! = 1 1! = 1 1 2! = 2 1 3! = 3 2 1 4! = 4 3 2 1 5! = 5 4 3 2 1 =1 =1 =2 =6 = 24 = 120

6! = 6 5 4 3 2 1 = 720 Summation The summation (also known as a series) is used more than almost any other technique in statistics. It is a method of representing addition over lots of values without putting + after +. We represent summation using a big uppercase sigma: . Examples Very often in statistics we will sum a list of related variables:

Here we are adding all the x variables (which will hopefully all have values by the time we calculate this). The expression below the (i=0, in this case) represents the index variable and what its starting value is (i with a starting value of 0) while the number above the represents the number that the variable will increment to (stepping by 1, so i = 0, 1, 2, 3, and then 4). Another example:

Notice that we would get the same value by moving the 2 outside of the summation (perform the summation and then multiply by 2, rather than multiplying each component of the summation by

2). Infinite series There is no reason, of course, that a series has to count on any determined, or even finite valueit can keep going without end. These series are called "infinite series" and sometimes they can even converge to a finite value, eventually becoming equal to that value as the number of items in your series approaches infinity (). Examples

This example is the famous geometric series. Note both that the series goes to (infinity, that means it does not stop) and that it is only valid for certain values of the variable r. This means that if r is between the values of -1 and 1 (-1 < r < 1) then the summation will get closer to (i.e., converge on) 1 / the series out. Linear Approximation Student-t Distribution at various critical values with varying degrees of freedom. v / 0.20 0.10 0.05 0.025 0.01 0.005
1-r

the further you take

40 0.85070 1.30308 1.68385 2.02108 2.42326 2.70446 50 0.84887 1.29871 1.67591 2.00856 2.40327 2.67779 60 0.84765 1.29582 1.67065 2.00030 2.39012 2.66028 70 0.84679 1.29376 1.66691 1.99444 2.38081 2.64790 80 0.84614 1.29222 1.66412 1.99006 2.37387 2.63869 90 0.84563 1.29103 1.66196 1.98667 2.36850 2.63157 100 0.84523 1.29007 1.66023 1.98397 2.36422 2.62589

Let us say that you are looking at a table of values, such as the one above. You want to approximate (get a good estimate of) the values at 63, but you do not have those values on your table. A good solution here is use a linear approximation to get a value which is probably close to the one that you really want, without having to go through all of the trouble of calculating the extra step in the table.

This is just the equation for a line applied to the table of data. xi represents the data point you want to know about, want to know about, and know about. is the known data point beneath the one you is the known data point above the one you want to

uppose you are sampling from a population with mean and standard deviation he sample size is n hat are the expected value and the variance of a sample mean ? ANS: If sample mean is xbar, E(xbar)==1065

Var(xbar) = (population variance)/n==100^2/100=100 OR

E[Xbar] = = 1,065 Var(Xbar) = ^2/n = 500^2/100 = 2500

Assignment Set- 1
Master of Business Administration-MBA Semester MB0041-Financial and Management Accounting Q.1 Assure you have just started a Mobile store. You sell mobile sets and currencies of Airtel, Vodaphone, Reliance and BSNL. Take five transactions and prepare a position statement after every transaction. Did you firm earn profit or incurred loss at the end? Make a small comment on your financial position at the end. ANS: We shall consider five transactions and show how they are accounted for in the
books of the business. 1. Mr. Rajesh brings Rs.100000 cash as capital into his business. 2. He purchases Mobile Set to his shop Rs.10000 3. He buys currencies for cash Rs.50000 4. He sells currencies worth Rs.30000 for Rs.40000 on credit to Arjun 5. He pays wages to servants Rs.1000 Transaction 1: The business receives capital in cash. Capital is a liability and cash is an asset to the business. Liability Asset Capital 100000 Cash 100000 Transaction 2: Mobile Set is purchased for cash. This transaction can be reflected as Under.

Q.2a. List the accounting standards issued by ICAI?

ANS: To bring uniformity in terminology, accounting concepts, conventions, and


assumptions, the Institute of Chartered Accountants of India (ICAI) established Accounting Standards Board (ASB) in 1977. An Accounting Standard is a selected set of accounting policies or broad guidelines. Example: While depreciating an asset the practice of adopting straight line method or diminishing balance method or any other method is a convention regarding the principles and methods to be chosen out of several alternatives. There are altogether 32 accounting standards issued by ASB out of which, one standard (AS8) has been withdrawn pursuant to AS26 becoming mandatory. 2b. Write short notes of IFRS?

ANS: IFRS are standards, interpretations and framework for the preparation and
presentation of financial statements. IFRS was framed by International Accounting Standards Board (IASB). The objective of financial statement is to provide information about the financial position, performance and changes in the financial position of an entity. It should also provide the current financial status of the entity to all the users of financial information. IFRS follows accrual basis of accounting and the financial statements are prepared on the basis that an entity will continue for the foreseeable future. IFRS helps entities access global capital market with ease. Q.3 Choose an Indian Company of your choice that has adopted Balance Score Card and detail on it?

ANS: The Balanced Score Card is a framework for integrating measures derived from
strategy. While retaining financial measures of past performance, the Balanced Score Card introduces the drivers of future financial performance. (Figure 1) The drivers (customer, internal business process, learning & growth perspectives) are derived from the organization's strategy translated into objectives and measures.

The Balanced Score Card is more than a measurement system it can be used as an organizing framework for their management processes. The real power of the Balanced Score Card is when it is transformed from a measurement system to a management system. It fills the void that exists in most management systems - the lack of a systematic process to implement and obtain feedback about strategy

Q.4 What is a cash budget? How it is useful in managerial decision making?

ANS: A proper control over cash is very essential. Cash is an important component in
any activity. The control becomes inescapable. If cash is not properly managed or if it is mismanaged, the ultimate result would be disastrous. In many times and in many business situations, business failures are noticed due to the lacunae found in the cash management. Hence cash budgeting occupies a pivotal place in the study of Financial Management.

Cash budgeting is the process of forecasting the expected receipts known as cash inflows, and expected payments known as cash outflows to meet the future obligations. The written statement of receipts and payments is known as the cash budget. It is a crystal ball which enables one to observe the future movements in cash position. It is a mere forecast of cash position of an undertaking for a definite period of time. The period may be daily, weekly, monthly, quarterly, semi-annually, or annually. The major two components of cash budget would be forecast first the cash receipts and then second forecasting the cash disbursements. The receipts of cash are formatted as follows: 1. Opening balance of cash in hand and cash at bank
The Management Accounting Perspective of the Business Enterprise The management accounting view of business may be divided into two broad categories: (1) basic features and (2) basic assumptions. Basic Features The business firm or enterprise is an organizational structure in which the basic activities are departmentalized as line and staff. There are three primary line functions: marketing, production, and finance. The organization is run or controlled by individuals collectively called management. The staff or advisory functions include accounting, personnel, and purchasing and receiving. The organization has a communication or reporting system (e.g. budgeting) to coordinate the interaction of the various staff and line departmental functions. The environment in which the organization operates includes investors, suppliers, governments (state and federal), bankers, accountants, lawyers, competitors, etc.) The organizational aspect of the business firm is illustrated in Figure 2.1. This descriptive model shows that there are different levels of management. A commonly used approach is to classify management into three levels: Top management, middle management, and lower level management. The significance of a hierarchy of management is that decision-making occurs at three levels. Basic Assumptions in Management Accounting

The framework of management accounting is based on a number of implied assumptions. Although no single work has attempted to identify all of the assumptions, . Five categories of assumptions will be presented: 1. Basic goals 2. Role of management 3. Nature of Decision-making 4. Role of the accounting department 5. Nature of accounting information Basic Goal Assumptions - The basic goals or objectives the business enterprise may be multiple. For example, the goal may be to maximize net income. Other goals could be to maximize sales, ROI, or earnings per share. Management accounting does not require a specific of type of goal. However, whatever form the goal takes, management will at all times try to achieve a satisfactory level of profit. A less than satisfactory level of profit may portend a change in management. Role of Management Assumptions - The success of the business depends primarily upon the skill and abilities of management-which skills can vary widely among different managers. The business is not completely at the mercy of market forces. Management can through its actions (decisions) influence and control events within limits. In order to achieve desired results, management makes use of specific planning and control concepts and techniques. Planning and control techniques which management may use include business budgeting, cost-volume-profit analysis, incremental analysis, flexible budgeting, segmental contribution reporting, inventory models, and capital budgeting models. Management, in order to improve decision-making and operating results, will evaluate performance through the use of flexible budgets and variance analysis. Decision-making Assumptions - A critical managerial function is decisionmaking. Decisions which management must make may be classified as marketing, production, and financial. Decisions may also be classified as strategic and tactical and long-run and short-run. A primary objective of decision-making is to achieve optimum utilization of the businesss capital or resources. Effective decision-making requires relevant information and special analysis of data. Accounting Department Assumptions - The accounting department is a primary

source of information necessary in making-decisions. The accounting department is expected to provide information to all levels of management. Management will consider the accounting department capable of providing data useful in making marketing, production, and financial decisions. Nature of Accounting Information - In order for the accounting department to make meaningful analysis of data, it is necessary to distinguish between fixed and variable costs and other types of costs that are not important in the recording of business transactions. Some but not all of the information needed by management can be provided from financial statements and historical accounting records. In addition to historical data, management will expect the management accountant to provide other types of data, such as estimates, forecasts, future data, and standards. Each specific 18 | CHAPTER TWO Management Accounting and Decision-Making managerial technique requires an identifiable type of information. The accounting department will be expected to provide the information required by a specific tool. In order for the accounting department to make many types of analysis, a separation of costs into fixed and variable will be required. The management accountant need not provide information beyond the relevant range of activity.

Q.5

State the importance of differentiating between the fixed costs and

variable costs in managerial decision?

ANS: Fixed Costs: These are those costs which remain fixed up to certain range of work
capacity no matter how much product you produce within that capacity range. Like factory building rent. You pay the rent no matter that did you use that building for making the products or not. Variable Costs: These are those costs which change with the change in the number of product units you produce. Like Material , Labor etc Mixed Cost/Semi Variable Costs: These are those cost the part of which is remain fixed and some part of the cost is variable.
Fixed vs. variable costs

Understanding the process of managing costs first requires an understanding of two general types of costs: fixed and variable. Fixed costs, or sunk costs as they are sometimes called, are those that generally do not vary between payment intervals. Generally, these costs cannot be altered on a short-term basis because of contractual agreements or simply because it is impractical.

Table 1: Examples of fixed and variable costs


Fixed monthly costs Rent... Yellow pages ad... Loan payment... Office managers salary... Doctors salary... Total fixed costs... Variable costs per patient visit Supplies (forms, covers, etc.)... Electricity to operate a roller table... Collections costs (stamps, invoices, etc.)... $3 $1 $1 $1,100 $400 $500 $3,000 $5,000 $10,000

ne way to determine your fixed costs is to consider the expenses you would continue to incur if you temporarily closed your practice and no patients were being treated. In this case, your rent, car leases, yellow page ad fees and loan payments would still be due. They generally do not change with increases or decreases in business activity. It is important to note that fixed costs are unvarying only within a certain range of business activity. For example: if the practice grows enough to require additional space or additional employees, the fixed costs associated with rent or salaries will change as well. Variable costs are those that change as the level of business activity changes. Examples of the variable costs within a chiropractic business would be supplies used for each patient visit, collection fees paid to external billing agencies and wages for hourly, part-time employees. These costs are driven primarily by the practices business activity, by number of patients that the practice treats.

The closed practice test we used above to determine the fixed costs of a practice can also be used to determine the variable costs. The variable costs are those that would stop if the practice were closed for a month and no patients were treated at all. Once you understand the difference between fixed and variable costs, it is important to know how to distinguish one from the other. For instance, consider a practice that has fixed costs of $10,000 and variable costs of $5 per patient. (See Table 1 for an example of fixed and variable costs.) To cover its monthly expenses, the practice would have to earn $10,000 in fees plus $5 per patient treated. If the practice had only one patient visit per month, it would have to charge $10,005 for that one treatment in order to cover its fixed and variable costs! If the practice had 1,000 patient visits during the month, its total costs would be $15,000 ($10,000 in fixed costs plus 1,000 patient visits at $5 each). Therefore, this practice would only have to charge $15 per patient visit to cover its fixed and variable costs.

Q.6 Identify the users of accounting information?

ANS: There are two types of users( internal and external):- list of
internal users:1. employees 2. management 3. shareholders/owners.

list of external users :those who have economic transactions like


suppliers creditors bankers financial institutions

others like

competitors government and regulatory agencies auditors researchers and academicians representatives of others interest like brokers ,underwriters etc

potential shareholders

* Management - obtain financial information in a way that best suits its needs, (i.e. ways to cut costs, maximize profit, etc.). * Board of Directors - information to assist in determination of current executive positions

External users: * IRS - requires the information be presented in a very specific manner. * Stockholders and potentional stockholders - information that will aid in decision continue to hold the stock, sell the stock, or purchase the stock. * Bondholders, bankers & other creditors - information that will aid in decision of whether or not to purchase the bond. Bank needs information that will help it determine the company's ability to repay the loan and interest. * Employee (can also be considered internal) - information that will aid in decision to continue working at the company or look for employment elsewhere. * Supplier - information to aid in decision to continue or start supplying to the company.

Master of Business Administration - MBA Semester I MB0042 - Managerial Economics Q.1 Income elasticity of demand has various applications. Explain each application with the help of an example. Ans :Income elasticity of demand may be defined as the ratio or proportionate change in the quantity demanded of a commodity to a given proportion change in the income. In short, it indicates the extent to which demand changes with a variation in consumers income. Thefollowing formula helps to measure the income elasticity (Ey). Or Where Ey is income elasticity of demand D is change in demand D is original demand Y is change in income Y is original income Example Original demand=400 units Original income= 4000 units New demand =700 units New income= 6000 units Change in demand= 700-400= 300 units change in income=6000-4000=2000

Hence Ey=300/2000*4000/400=1.5 Generally speaking Ey is positive. This is because there is a direct relationship between income and demand, i.e. higher the income; higher would be the demand and vice versa. On the basis of the numerical value of the co-efficient, Ey is classified as greater than one, less than one, equal to one, equal to zero and negative. The concept of ey helps us in classifying commodities in to different categories.

1. When Ey is positive, the commodity is normal (used in day-to-day life) 2. When Ey is negative, the commodity is inferior. ( for example jowar, beedi etc) 3. When Ey is positive and greater than one, the commodity is luxury. 4. When Ey is positive but less than one, the commodity is essential. 5. When Ey is zero, the commodity is neutral. E.g. salt, match box etc. Practical application of income elasticity of demand 1. Helps in determining the rate of growth of the firm. If the growth rate of the economy and income growth of the people is reasonable forecasted, in that case it is possible predict expected increase in the sales of a firm and vice versa.

2. Helps in the demand forecasting of a firm. It can be in estimating future demand provided the rate of increase in income and Ey for the products are known. Thus, it helps in demand forecasting activities of a firm. 3. Helps in production planning and marketing. The knowledge of Ey is essential for production planning, formulating marketing strategy, deciding advertising expenditures and nature of distribution channel etc in the long run. 4. Helps in ensuring stability in production. Proper estimation of different degrees of income elasticity of demand for different types of product helps in avoiding over-production or under-production of a firm. One should know whether rise or fall in income is permanent or temporary. 5. Helps in estimating construction of houses. The rate of growth in incomes of people also helps in housing programs in a country. Thus it helps a lot in managerial decisions of a firm. Q.2 When is the opinion survey method used and what is the effectiveness of the method. Ans :Survey of buyers intention or preference is one of the important methods of demand forecasting. It is also called Opinion Survey Method. Under this method, consumer buyers are requested to indicate their preference and

willingness about a particular product. They are about to reveal their future purchase plans with respect to specific items. They are expected to give answer to question like what items they intends to buy, in what quantity, why, where, what quality they expect, how much they are planning to spend etc. Generally, the field surveys are conducted by the marketing research departments of the company or hiring the services of outside research organization consisting of learned and highly qualified professionals. The heart of the survey is questionnaire. It is a comprehensive one covering almost all questions either directly or indirectly in a most intelligent manner. It is prepared by an expert body who are specialist in the field or marketing. The questionnaire is distributed among the consumer either through mail or in person by the company. Consumers are requested to furnish all relevant and correct information. The next step is to collect the questionnaire from the consumers for the purpose of evaluation. The materials collected will be classified, edited and analyzed. If any bias prejudices, exaggerations, artificial or excess demand creation are found at the time of answering they would be eliminated. The information so collected will now be consolidated and reviewed by the top executives with lot of experiences. It will be examined thoroughly. Inferences are drawn and conclusions are arrived at. Finally a report is prepared and submitted to the management for taking final decisions. The success of the survey method depends on many factors:

1. The nature of the question asked. 2. The ability of the surveyed. 3. The representative of the sample 4. Nature of the product 5. Characteristics of the market 6. Consumer behavior 7. Techniques of analysis 8. Conclusion drawn etc. The management should not entirely depend on the result of survey reports t project future demand. Consumer may not express their honest and real views and as such they may give only the broad trends in the market. In order to arrive, at right conclusion, field surveys should be regularly checked and supervised. This method is simple and useful to the producers who produce goods in bulk. Here the burden of forecasting is put on the customers. However this method is not much useful in estimating the future demand of the household as they run in a large numbers and also do not freely express their future demand requirements. It is expensive and so difficult. Preparation of questionnaire is not an easy task. At best it can be used for short term forecasting.

Q.3 Show how price is determined by the forces of demand and supply, by using forces of equilibrium. Ans The word equilibrium is derived from the Latin word aequilibrium which means equal balance. It means a state of even balance in which opposing forces or tendencies neutralize each other. It is a position of rest characterized by absence of change. It is a state where there is complete agreement of the economic plans of the various market participants so that no one has a tendency to revise or alter his decision. In the words of professor Mehta: Equilibrium denotes in economics absence of change in movement. Market Equilibrium There are two approaches to market equilibrium viz., partial equilibrium approach and the general equilibrium approach. The partial equilibrium approach to pricing explains price determination of a single commodity keeping the prices of other commodities constant. On the other hand, the general equilibrium approach explains the mutual and simultaneous determination of the prices of all goods and factors. Thus it explains a multi market equilibrium position. Earlier to Marshall, there was a dispute among economists on whether the force of demand or the force of supply is more important in determining price. Marshall gave equal importance to both demand and supply in the determination of value or price. He compared supply and demand to a pair of scissors - We might as reasonably dispute whether it is the upper or the under blade of a pair of scissors that cuts a piece of paper, as whether value is governed by utility or cost of production. Thus neither the upper blade nor the lower blade taken separately can

cut the paper; both have their importance in the process of cutting. Likewise neither supply alone, nor demand alone can determine the price of a commodity, both are equally important in the determination of price. But the relative importance of the two may vary depending upon the time under consideration. Thus, the demand of all consumers and the supply of all firms together determine the price of a commodity in the market. Equilibrium between demand and supply price: Equilibrium between demand and supply price is obtained by the interaction of these two forces. Price is an independent variable. Demand and supply are dependent variables. They depend on price. Demand varies inversely with price, a rise in price causes a fall in demand and a fall in price causes a rise in demand. Thus the demand curve will have a downward slope indicating the expansion of demand with a fall in price and contraction of demand with a rise in price. On the other hand supply varies directly with the changes in price, a rise in price causes a rise in supply and a fall in price causes a fall in supply. Thus the supply curve will have an upward slope.At a point where these two curves intersect with each other the equilibrium price is established. At this price quantity demanded is equal to the quantity demanded. This we can explain with the help of a table and a diagram Price in rs 30 25 20 10 5 Demand in units 5 10 15 20 30 25 20 15 10 5 Supply in units State of market D<S D<S D=S D>S D>S Pressure on price P decreases P decreases Neutral P increases P increases

In the table at Rs.20 the quantity demanded is equal to the quantity supplied. Since the price is agreeable to both the buyer and sellers, there will be no tendency for it to change; this is called equilibrium price. Suppose the price falls to Rs.5 the buyer will demand 30 units while the seller will supply only 5 units. Excess of demand over supply pushes the price upward until it reaches the equilibrium position supply is equal to the demand. On the other hand if the price rises to Rs.30 the buyer will demand only 5 units while the sellers are ready to supply 25 units. Sellers compete with each other to sell more units of the commodity. Excess of supply over demand pushes the price downward until it reaches the equilibrium. This process will continue till the equilibrium price of Rs.20 is reached. Thus the

interactions of demand and supply forces acting upon each other restore the equilibrium position in the market. In the diagram DD is the demand curve, SS is the supply curve. Demand and supply are in equilibrium at point E where the two curves intersect each other. OQ is the equilibrium output. OP is the equilibrium price. Suppose the price OP2 is higher than the equilibrium price OP. at this point price quantity demanded is P2D2. Thus D2S2 is the excess supply which the seller wants to push into the market, competition among the sellers will bring down the price to the equilibrium level where the supply is equal to the demand. At price OP1, the buyers will demand P1D1 quantity while the sellers are ready to sell P1S1. Demand exceeds supply. Excess demand for goods pushes up the price; this process will go until equilibrium is reached where supply becomes equal to demand. Q.4 Distinguish between fixed cost and variable cost using an example. Ans: Fixed cost: These costs are incurred on fixed factors like land, building, equipments, plants, superior types of labour, top management etc. fixed costs in the short run remains constant because the firm does not change the size of plant and the amount of the fixed factors employed. Fixed costs do not vary with either expansion or contraction in output. These cost are to be incurred by a firm even output is zero. Even if the firm close down its operation for some time temporarily in the short run, but remains in business, these cost have to be borne by it. Hence, these costs are independent of output and are referred to as unavoidable contractual cost. Prof. Marshall called fixed cost as supplementary costs. They include such items as

contractual rent payments, interest on capital borrowed, insurance premium, depreciation and maintenance allowance, administrative expenses like managers salary or salary of the permanent staff, property and business taxes, license fees, etc. They are called as over- head costs because these costs are to incurred whether there is production or not. These costs are to be distributed on each units of output produced by a firm. Hence, they are called as indirect costs. Variable Costs: The costs corresponding to variable factors are described as variable costs. These costs are incurred on raw materials, ordinary labour, transport, power, fuel, water etc, which directly vary in the short runs. Variable costs are directly and proportionately increases or decreases with the level of output. If a firm shut down for some times in the short run; then it will not use the variable factors of production and will not therefore incurs any variable costs. Variable costs are incurred only when some amount of output is produced. Total variable cost increases with the level of increase in the level of production and vice-versa. Prof. Marshall called variable costs as prime costs or direct costs because the volume of output produced by a firm depends directly upon them. It is clear from the above description that a production cost consists of both fixed as well as variable costs. The difference between the two is meaningful and relevant only in the short run. In the long run all costs become variable because all factors of production become adjustable and variable in the long run. However, the distinction between the fixed and variable costs is very important in theshort because it influences the average costs behavior of the firm. In the short run, even if a firm wants to close down its operation but wants to remain in the business, it will have to incur fixed costs but it must cover at least its variable costs.

Q.5 Discuss Marris Growth Maximization model ? Profit maximization is traditional objective of a firm. Sales maximization objective is explained by Prof. Boumal. On similar lines, Prof. Marris has developed another alternative growth maximization model in recent years. It is a common factor to observe that each firm aims at maximizing its growth rate as this goal would answer many of the objectives of a firm. Marris points out that a firm has to maximize its balanced growth rate over a period of time. Marris assumes that the ownership and control of the firm is in the hands of two groups of people, i.e. owner and managers. He further points out that both of them have two distinctive goals. Managers have a utility function in which the amount of salary, status, position, power, prestige and security of job etc are the most import variable where as in case of are more concerned about the size of output, volume of profits, market shares and sales maximization. Utility function of the manager and that the owner are expressed in the following mannerUo= f [size of output, market share, volume of profit, capital, public esteem etc.] Um= f [salaries, power, status, prestige, job security etc.] In view of Marris the realization of these two functions would depend on the size of the firm. Larger the firm, greater would be the realization of these functions and vice-versa. Size of the firm according to Marris depends on the amount of corporate capital which includes total volume of the asset, inventory level, cash reserve etc. He

further points out that the managers always aim at maximizing the rate of growth of the firm rather than growth in absolute size of the firms. Generally managers like to stay in a grouping firm. Higher growth rate of the firm satisfy the promotional opportunity of managers and also the share holders as they get more dividends.

Q.6 Explain how fiscal policy is used to achieve economic stability. Ans: In order to achieve a stable economic condition, fiscal policy has to play a positive and constructive role both in developed and developing nations. The specific role to be played by fiscal policy can be discussed as follows: To act as optimum allocator of resources: As most of the resources are scarce in their supply, careful planning is needed in its allocation so as to achieve the set targets. Rational allocation would ensure fulfillment of various objectives. To act as a saver: 1. It should follow a rational consumption policy reduces the MPC and raises the MPS. 2. Taxation policy has to be modified to raise the rates of old taxes, introduces new additional taxes, and extends the tax-nets. 3. Profit earning capacity of public sector units are to be raise substantially to mopup financial resources. 4. The government should borrow more money both in the country and outside the country.

5. Higher the rate of interest are to be offered for government bonds and security. To act as an investor: Mere mobilization of financial resources is not an end in itself. It should result in the creation of real resources which are more important in accelerating the growth process. Rapid economic growth depends upon the volume of investment. Hence, fiscal policies have to be ensuring higher volume of investment in both private and public sectors. To act as price stabilizer: price stability is of paramount of importance in an economy.Extreme levels of both inflation and deflation would disrupt and disturb the normal and regular working of an economic system. This would come in the way of stable and persistent growth. Hence all measures are to be taken to check these two dangerous situations so as to create necessary congenial atmosphere to prepare the background for rapid economic growth. To act as an economic stabilizer: Price stability would create the necessary background for over all economics stability. Upswing and downswing in the level of economic activities are to be avoided. If an economy is subject to frequent fluctuation in the form of trade cycle, certainly, it would undermine and disturb the growth process. Instability would come in the way of persistent and consistent growth in a country. Hence all measure to be taken to ensure economic stability. To act as an employment generator: Fiscal policy should help in mobilizing more financial resources, convert them in to investment and create more employment opportunity to absorb the huge unemployed man power. To act as balancer: There must be proper balance between aggregate saving and

aggregate investment, demand and supply, income and output and expenditure, economic overhead capital and social overhead capital etc. Any sort of imbalance would result in either surpluses or scarcity in different sectors of the economy leading to fast growth in some sectors followed by lagging of some other sectors. To act as growth promoter: The basic objective of any economic policy is to ensure higher economic growth rates. This is possible when there is higher national savings, investment, production, employment and income. Hence, fiscal policy is to be designed in such a manner so as to promote higher growth in an economy. To act as in come redistribute: Fiscal policy has to minimize inequalities and ensure distributive justice in an economy. This is possible when a rational taxation and public expenditure policy is adopted. More money is collected from richer section of the society through various imaginative taxation policies and a larger amount of money is to be spent in favor of poorer sections of the society. Thus, inequality is to be reduced to the minimum. To act as stimulator of living standards of people: the final objective is to raise the level of living standards of the people. This is possible when there is higher output, income and employment leading to higher purchasing power in the hands of common man. Hence, fiscal policy should help in creating more wealth in the economy. If there is economic prosperity, then it is possible to have a satisfactory, contended and peaceful life. Thus, fiscal policy has to play a major role in promoting economic growth in a country.

Master of Business Administration-MBA Semester I Subject Code - MB0043 Subject Name - Human Resource Management Q.1 Write down the difference between Personnel management and Human Resource management.[10 Marks] Ans Personnel management is workforce centred, directed mainly at the organizations employees; such as finding and training them, arranging for them to be paid, explaining managements expectations, justifying managements actions etc. While on the other hand, HRM is resource -centered, directed mainly at management, in terms of devolving the responsibility of HRM to line management, management development etc. Although indisputably a management function, personnel management has never totally identified with management interests, as it becomes ineffective when not able to understand and articulate the aspirations and views of the workforce, just as sales representatives have to understand and articulate the aspirations of the customers. Personnel Management is basically an operational function, concerned primarily with carrying out the day-to day people management activities. While on the other hand, HRM is strategic in nature, that is, being concerned with directly assisting an organization to gain sustained competitive advantage.

HRM is more proactive than Personnel Management. Whereas personnel management is about the maintenance of personnel and administrative systems, HRM is about the forecasting of organizational needs, the continual monitoring and adjustment of personnel systems to meet current and future requirements, and the management of change. Q.2 Write a note on scope of HR in India. [10 Marks] Ans Its only in the past 10-12 years with the immense growth on account of the IT

industry that winds of change began to blow. It was largely the advent of the Information Technology era in India that brought with it the western management practices. MNCs (multinational companies) started up their operations in India. The FDI (foreign direct investment) went up steeply as the world saw the potential in the countrys human resources. India became a preferred location for MNCs primarily from the USA, followed by other developed countries. The gave birth to a new generation of management as well as HRM practices. New hiring methods, new ways of paying salaries, new employment terms and most importantly increased focus on individual performance and outcomes. There was emphasis on deliverables and linking individual and team performance to business results and success. Given the highly educated workforce there was a de-emphasis in the role of the trade unions. The era of the trade union dominance gave way to the new order of individual negotiated salaries and terms and clearly performance linked assessment systems. Another transformation that the Indian workplace witnessed was the focus on ethics and ethical practices in doing business. It was only fair to expect that with the weak legal system, it needed the support of the government policies and the corporate policies to beat the corruption that existed.

This has significantly contributed to India emerging as a preferred destination for doing business. All of this has yielded to give way to the birth of the professional manager Professional managers today are a critical and essential part of the Indian corporate. The professional manager brought about a shift in the culture from a highly authoritarian approach of getting work done, to a more collaborative and participative approach. The entrepreneurs who earlier operated in a secure, sheltered market and hardly face challenges, were challenged by the globalization that swept in with the liberalization policies and measure brought in by the Indian government late 1995 and onwards. Despite the challenges, the Indian employee and his manager evolved. Together they stepped up to face the challenge head-on and to win not only in India but also globally. The levers of (a) low cost, (b) highly skilled, and (c) English as the medium of education and it being the corporatelanguage: were the key drivers that enabled the flow of global business to India. There was exponential growth in employment both directly (jobs in the international and domestic companies) as well as indirectly (as support industries like transport, catering and ancillary industries). The simultaneous investment of the government in building the necessary infrastructure did its share of providing impetus towards creating more jobs for the people of the country. Hence, human relations movement in India has evolved very differently as compared to what we see in the developed economies of the USA and the UK. What is currently acting as a limitation is the enhanced awareness on the need for research based HRM practices. While there is a lot of work happening in the Indian education system to promote this, it is going to take a while before it can create a distinct body of knowledge that is referenceable. For now the industry relies on emulating westerns HR practices and customizing on a as-needed basis for the Indian corporation. For the rest the industry forums and consortiums like the

NASSCOM act as a hub bringing together organizations on a regular basis to discuss challenges and share best practices and identify ways and means o overcome them together. So far this has been successful and working to the advantage of the Indian corporate. Leading MNC research and consulting firms like Mercer and Hewitt too contribute to the industry through carrying out research and sharing reports on a regular basis. The approach however remains analytical and less prescriptive. Q.3 Explain the critical steps in Human Resource Planning system .[10 Marks] Ans Human Resource planning can be defined as a process by which an organization ensures thatit has the right number and kinds of people, at the right place, at the right time, capable ofeffectively and efficiently completing those tasks that will help the organization achieve itsoverall objectives or in other words HRP can be defined as planning for the future personnelneeds of an organization, taking into account both internal activities and factors in theexternal environment. Need and Importance of HRP Human resource Planning translates the organization objectives and plans into the number of workers needed to meet these objectives. The need and importance of HRP is as follows: determining the future manpower requirements and avoids problems like over staffing or understaffing in the organization. policies etc. that generates changes in the job content, skill requirements and number and types of personnel required.

staff becomes redundant, the HR manager has to attract and retain qualified and skilledpersonnel and also required to deal with issues like career development, successionplanning for which he takes the help of HRP. employment and promotional appointments to the candidates fro weaker sections, physically handicapped and socially and politically oppressed citizens. execution of personnel functions like recruitment, selection, transfers, promotions, layoffs, training and development and performance appraisal. turn will facilitate reduction in personal costs. assignments to develop managers and to ensure the organization has a steady supply of experienced and skilled employees. Factors affecting Human Resource Planning HRP is a dynamic and on going process. The process of updating is not very simple, since HRP is influenced by many factors, which are as follows: of staff needed. adopted by it. For e.g. the growth of a business calls for hiring of additional labour, while mergers will need a plan for layoffs.

carefully formulate the HR policies and so the HR manager has to evolve suitablemechanism to deal with uncertainties through career developments, successionplanning, retirement schemes etc. -term plans are adopted. And this time span is based on the degree of environmental uncertainties. factor influencing HRP. Accurate and timely human resource information system helps in getting better quality personnel. ensure that suitable candidates should be appointed at the right kind of job. So these are some of the factors that affect the human resource planning.

Limitations of Human Resource Planning s of an organization, as future is always uncertain. resources. approach and at the same time are conscious about the changing environment.

turnover

Q.4 List and explain the sources of recruitment? [10 Marks] Ans : The sources of employees recruitment can be classified into two types,internal and external. Filing a job opening from within the firm has advantages of stimulating preparation for possible transfer of promotion, increasing the general level of morale, and providing moreinformation about job candidates through analysis of work histories within the organisation.A job posting has number of advantages. From the view point of employee, it providesflexibility and greater control over career progress. The jobs posted on notice boards, group emails and publishing in internal news papers. Internal applications often restricted to certain employees, the guidelines for companies include: -Good or better in most recent performance review -Dependable attendance record -Not under probationary sanction -Having been in present position for at least one year The personnel department acts as a clearing house in screening applications that areunrealistic, preventing an excess number of bids by a single employee and counsellingunsuccessful employees in their constant attempt to change their jobs. Similarly the firm may go to external sources for recruitment of lower entry jobs, forexpansion, and positions whose specifications cannot be met by the present personnel. Thefollowing external resources are available for firms: a)Advertising: There is a trend toward more selective recruitment in advertising. This canbe affected in at least two ways. First advertisements can be placed in media read only byparticular groups. Secondly, more information about the company, the job, and the jobspecification can be included in the ad to permit some self-screening.

b)Employment Agencies: Additional screening can be affected through the utilization of employment agencies, both public and private. Today, in contrast to their formerunsavoury reputation, the public employment agencies in several states are wellregarded,particularly in the fields of unskilled semi-skilled and skilled operative jobs. In thetechnical and professional areas, however, the private agencies tend to specialize in aparticular engineer. c)Employee Referrals: Friends and relatives of present employees are also a good source from which employees may be drawn. When the labour market is very tight, largeemployers frequently offer their employees bonus or prizes for any referrals that are hiredand stay with the company for a specific length of time. Some companies maintain aregister of former employees whose record was good to contact them when there are newjob openings for which they are qualified. This method of recruitment, however, suffersfrom a serious defect that it encourages nepotism, i.e.Persons of ones community orcaste are employed, who may or may not be fit for the job. d)Schools, colleges and professional institutions: These offer opportunities for recruiting their students. They will also have separate placement cell where the bio data and otherparticulars of the students are available. The prospective employers can reviewcredentials and interview candidates for management trainees or probationers. This is anexcellent source of potential employees for entry-level positions in the organisations e)Labour Unions: Firms which closed or union shops must look to the union in their recruitment efforts. This has disadvantage of monopolistic workforce. f)Casual applicants: Unsolicited applications, both at the gate and through the mail, constitute a much-used source of personnel. These can be developed through

attractive employment office facilities, prompt and courteous reply to unsolicited letters. g)Professional organisations or recruiting firms or executive recruiters:Maintain complete records about employed executives. These firms are looked upon as headhunters, raiders and pirates by organizations may employ executive search firms to helpthem find talent. These consulting firms recommend persons of high calibre formanagerial, marketing and production engineers posts. h)Indoctrination seminars for colleges are arranged to discuss the problem of companies and employees. Professors are invited to take part of these seminars. Visits to plants arearranged so that professors may be favourably impressed. They may speak well of acompany and help it in getting the required personnel. i)Unconsolidated applications: for positions in which large numbers of candidates are notavailable from other sources, the companies may gain keeping files of applicationsreceived from candidates who make direct enquiries about the possible vacancies on theirown, or may send unsolicited applications. This would be helpful to firms for futurevacancies. j)Nepotism:the hiring of relatives will be an inevitable component of recruitment programmes in family-owned firms, such a policy does not necessarily coincide withhiring on the basis of merit, but interest and loyalty to the enterprise are offsettingadvantages. k)Leasing: to adjust to short term fluctuations in personnel needs, the possibility of leasingpersonnel by the hour or day should be considered. This principle has been particularlywell developed in the office administration field because they can avoid any obligation inpensions, insurance and any other fringe benefits.

l)Voluntary Organisations: Such as private clubs, social organisations might also provideemployees - handicaps, widowed or married women, old persons, retired hands etc. Inresponse to advertisements. m)Computer Data Banks: when a company desires a particular type of employees, jobspecifications and requirements are fed to computers, where they are matched againstdata stored in. This method is very useful in identifying candidates for hard-t-fit positionswhich calls for unusual combinations of skills. Q.5 Describe the grievance handling procedure. Ans Handling a grievance When dealing with a grievance:

ensure you're familiar with the procedure and apply it correctly hold any grievance hearing in private without interruptions where a grievance relates to the person's line manager, ensure that the employee can raise the grievance with someone else

listen carefully to the person's explanation of the problem and consider whether there is a deeper issue which might be the root cause of the grievance

listen to any conflicting points of view weigh up all evidence to see whether there is an issue you need to address

decide what action to take, trying to balance fairness to the person without compromising the business or other workers

inform all concerned parties of your decision and the appeal process

ensure you resolve any problems relating to policies, procedures or conduct where the grievance procedure highlights these

keep the process as confidential as possible

Deal with grievances sensitively, particularly where they concern other workers. You may wish to develop specific procedures for very sensitive matters involving unfair treatment eg, discrimination, bullying or harassment. . Consider also having a separate "whistleblowing" procedure, so that workers are encouraged to raise any complaints about wrongdoing eg fraud, internally rather than disclosing them outside the business. If a worker raises a separate grievance during a disciplinary hearing, it's good practice to adjourn the hearing until the grievance is dealt with. By dealing with problems in a fair and reasonable manner, you're much less likely to lose valued and skilled staff through resignation. It will also help you successfully defend a constructive dismissal claim. Q 6 what are objectives of training? Ans Training objectives are one of the most important parts of training program. While some people think of training objective as a waste of valuable time. The counterargument here is that resources are always limited and the training objectives actually lead the design of training. It provides the clear guidelines and develops the training program in less time because objectives focus specifically on needs. It helps in adhering to a plan.

Training objective tell the trainee that what is expected out of him at the end of the training program. Training objectives are of great significance from a numberofstakeholderperspectives, 1.Trainer 2.Trainee 3.Designer 4.Evaluator Trainer - The training objective is also beneficial to trainer because it helps the trainer to measure the progress of trainees and make the required adjustments. Also, trainer comes in a position to establish a relationship between objectives and particular segments of training.

Trainee - The training objective is beneficial to the trainee because it helps in reducing the anxiety of the trainee up to some extent. Not knowing anything or going to a place which is unknown creates anxiety that can negatively affect learning. Therefore, it is important to keep the participants aware of the happenings, rather than keeping it surprise.

Secondly, it helps in increase in concentration, which is the crucial factor to make the training successful. The objectives create an image of the program in trainees mind that actually helps in gaining attention. Thirdly, if the goal is set to be challenging and motivating, then the likelihood of achieving those goals is much higher than the situation in which no goal is set. Therefore, training objectives helps in increasing the probability that the participants will be successful in training. training

Designer - The training objective is beneficial to the training designer because if the designer is aware what is to be achieved in the end then hell buy the training package according to that only. The training designer would then look for the training methods, training equipments, and training content accordingly to achieve those objectives. Furthermore, planning always helps in dealing effectively in an unexpected situation. Consider an example; the objective of one training program is to deal effectively with customers to increase the sales. Since the objective is known, the designer will design a training prog that will include ways to improve the interpersonal skills, such as verbal and non verbal language, dealing in unexpected situation i.e. when there is a defect in a product or when a customer

angry. Therefore, without any guidance, the training may not be designed appropriately. Evaluator - It becomes easy for the training evaluator to measure the progress of the trainees because the objectives define the expected performance of trainees. Training objective is an important to tool to judge the performance of participants.

S-ar putea să vă placă și